Slot System
Featured Buckets
Featured Buckets Admin

Which anticoagulant is safest for frail elderly patients with nonvalvular A-fib?

Article Type
Changed
Wed, 11/16/2022 - 16:01
Display Headline
Which anticoagulant is safest for frail elderly patients with nonvalvular A-fib?

ILLUSTRATIVE CASE

A frail 76-year-old woman with a history of hypertension and hyperlipidemia presents for evaluation of palpitations. An in-office electrocardiogram reveals that the patient is in AF. Her CHA2DS2-VASc score is 4 and her HAS-BLED score is 2.2,3 Using shared decision making, you decide to start medications for her AF. You plan to initiate a beta-blocker for rate control and must now decide on anticoagulation. Which oral anticoagulant would you prescribe for this patient’s AF, given her frail status?

Frailty is defined as a state of vulnerability with a decreased ability to recover from an acute stressful event.4 The prevalence of frailty varies by the measurements used and the population studied. A 2021 meta-analysis found that frailty prevalence ranges from 12% to 24% worldwide in patients older than 50 years5 and may increase to > 30% among those ages 85 years and older.6 Frailty increases rates of AEs such as falls7 and fracture,8 leading to disability,9 decreased quality of life,10 increased utilization of health care,11 and increased mortality.12 A number of validated approaches are available to screen for and measure frailty.13-18

Given the association with negative health outcomes and high health care utilization, frailty is an important clinical factor for physicians to consider when treating elderly patients. Frailty assessment may allow for more tailored treatment choices for patients, with a potential reduction in complications. Although CHA2DS2-VASc and HAS-BLED scores assist in the decision-making process of whether to start anticoagulation,these tools do not take frailty into consideration or guide anticoagulant choice.2,3 The purpose of this study was to analyze how levels of frailty affect the association of 3 different direct oral anticoagulants (DOACs) vs warfarin with various AEs (death, stroke, or major bleeding).

STUDY SUMMARY

This DOAC rose above the others

This retrospective cohort study compared the safety of 3 DOACs—dabigatran, rivaroxaban, and apixaban—vs warfarin in Medicare beneficiaries with AF, using 1:1 propensity score (PS)–matched analysis. Eligible patients were ages 65 years or older, with a filled prescription for a DOAC or warfarin, no prior oral anticoagulant exposure in the previous 183 days, a diagnostic code of AF, and continuous enrollment in Medicare Parts A, B, and D only. Patients were excluded if they had missing demographic data, received hospice care, resided in a nursing facility at drug initiation, had another indication for anticoagulation, or had a contraindication to either a DOAC or warfarin.

Frailty was measured using a claims-based frailty index (CFI), which applies health care utilization data to estimate a frailty index, with cut points for nonfrailty, prefrailty, and frailty. The CFI score has 93 claims-based variables, including wheelchairs and durable medical equipment, open wounds, diseases such as chronic obstructive pulmonary disease and ischemic heart disease, and transportation services.15-17 In this study, nonfrailty was defined as a CFI < 0.15, prefrailty as a CFI of 0.15 to 0.24, and frailty as a CFI ≥ 0.25.

Among older patients treated with anticoagulation for atrial fibrillation, apixaban had the lowest adverse event rate vs warfarin among frail patients, compared with dabigatran and rivaroxaban.

The primary outcome—a composite endpoint of death, ischemic stroke, or major bleeding—was measured for each of the DOAC–warfarin cohorts in the overall population and stratified by frailty classification. Patients were followed until the occurrence of a study outcome, Medicare disenrollment, the end of the study period, discontinuation of the index drug (defined as > 5 days), change to a different anticoagulant, admission to a nursing facility, enrollment in hospice, initiation of dialysis, or kidney transplant. The authors conducted a PS-matched analysis to reduce any imbalances in clinical characteristics between the DOAC- and warfarin-­treated groups, as well as a sensitivity analysis to assess the strength of the data findings using different assumptions.

The authors created 3 DOAC–warfarin cohorts: dabigatran (n = 81,863) vs warfarin (n = 256,722), rivaroxaban (n = 185,011) vs warfarin (n = 228,028), and apixaban (n = 222,478) vs warfarin (n = 206,031). After PS matching, the mean age in all cohorts was 76 to 77 years, about 50% were female, and 91% were White. The mean HAS-BLED score was 2 and the mean CHA2DS2-VASc score was 4. The mean CFI was 0.19 to 0.20, defined as prefrail. Patients classified as frail were older, more likely to be female, and more likely to have greater comorbidities, higher scores on CHA2DS2-VASc and HAS-BLED, and higher health care utilization.

Continue to: In the dabigatran-warfarin...

 

 

In the dabigatran–warfarin cohort (median follow-up, 72 days), the event rate of the composite endpoint per 1000 person-years (PY) was 63.5 for dabigatran and 65.6 for warfarin (hazard ratio [HR] = 0.98; 95% CI, 0.92 to 1.05; rate difference [RD] per 1000 PY = –2.2; 95% CI, –6.5 to 2.1). A lower rate of the composite endpoint was associated with dabigatran than warfarin for the nonfrail subgroup but not the prefrail or frail groups.

In the rivaroxaban–warfarin cohort (median follow-up, 82 days), the composite endpoint rate per 1000 PY was 77.8 for rivaroxaban and 83.7 for warfarin (HR = 0.98; 95% CI, 0.94 to 1.02; RD per 1000 PY = –5.9; 95% CI, –9.4 to –2.4). When stratifying by frailty category, both dabigatran and rivaroxaban were associated with a lower composite endpoint rate than warfarin for the nonfrail population only (HR = 0.81; 95% CI, 0.68 to 0.97, and HR = 0.88; 95% CI, 0.77 to 0.99, respectively).

In the apixaban–warfarin cohort (median follow-up, 84 days), the rate of the composite endpoint per 1000 PY was 60.1 for apixaban and 92.3 for warfarin (HR = 0.68; 95% CI, 0.65 to 0.72; RD per 1000 PY = –32.2; 95% CI, –36.1 to –28.3). The beneficial association for apixaban was present in all frailty categories, with an HR of 0.61 (95% CI, 0.52 to 0.71) for nonfrail patients, 0.66 (95% CI, 0.61 to 0.70) for prefrail patients, and 0.73 (95% CI, 0.67 to 0.80) for frail patients. Apixaban was the only DOAC with a relative reduction in the hazard of death, ischemic stroke, or major bleeding among all frailty groups.

WHAT’S NEW

Only apixaban had lower AE rates vs warfarin across frailty levels

Three DOACs (dabigatran, rivaroxaban, and apixaban) reduced the risk of death, ischemic stroke, or major bleeding compared with warfarin in older adults with AF, but only apixaban was associated with a relative reduction of these adverse outcomes in patients of all frailty classifications.

CAVEATS

Important data but RCTs are needed

The power of this observational study is considerable. However, it remains a retrospective observational study. The authors attempted to account for these limitations and potential confounders by performing a PS-matched analysis and sensitivity analysis; however, these findings should be confirmed with randomized controlled trials.

Continue to: Additionally, the study...

 

 

Additionally, the study collected data on each of the DOAC–warfarin cohorts for < 90 days. Trials to address long-term outcomes are warranted.

Finally, there was no control group in comparison with anticoagulation. It is possible that choosing not to use an anticoagulant is the best choice for frail elderly patients.

CHALLENGES TO IMPLEMENTATION

Doctors need a practical frailty scale, patients need an affordable Rx

Frailty is not often considered a measurable trait. The approach used in the study to determine the CFI is not a practical clinical tool. Studies comparing a frailty calculation software application or an easily implementable survey may help bring this clinically impactful information to the hands of primary care physicians. The Clinical Frailty Scale—a brief, 7-point scale based on the physician’s clinical impression of the patient—has been found to correlate with other established frailty measures18 and might be an option for busy clinicians. However, the current study did not utilize this measurement, and the validity of its use by primary care physicians in the outpatient setting requires further study.

Cost may be a barrier for patients younger than 65 years or for those older than 65 years who do not qualify for Medicare or do not have Medicare Part D.

In addition, cost may be a barrier for patients younger than 65 years or for those older than 65 years who do not qualify for Medicare or do not have Medicare Part D. The average monthly cost of the DOACs ranges from $560 for dabigatran19 to $600 for rivaroxaban20 and $623 for apixaban.21 As always, the choice of anticoagulant therapy is a clinical judgment and a joint decision of the patient and physician.

Files
References

1. Kim DH, Pawar A, Gagne JJ, et al. Frailty and clinical outcomes of direct oral anticoagulants versus warfarin in older adults with atrial fibrillation: a cohort study. Ann Intern Med. 2021;174:1214-1223. doi: 10.7326/M20-7141

2. Zhu W, He W, Guo L, et al. The HAS-BLED score for predicting major bleeding risk in anticoagulated patients with atrial fibrillation: a systematic review and meta-analysis. Clin Cardiol. 2015;38:555-561. doi: 10.1002/clc.22435

3. Olesen JB, Lip GYH, Hansen ML, et al. Validation of risk stratification schemes for predicting stroke and thromboembolism in patients with atrial fibrillation: nationwide cohort study. BMJ. 2011;342:d124. doi: 10.1136/bmj.d124

4. Xue QL. The frailty syndrome: definition and natural history. Clin Geriatr Med. 2011;27:1-15. doi: 10.1016/j.cger.2010.08.009

5. O’Caoimh R, Sezgin D, O’Donovan MR, et al. Prevalence of frailty in 62 countries across the world: a systematic review and meta-analysis of population-level studies. Age Ageing. 2021;50:96-104. doi: 10.1093/ageing/afaa219

6. Campitelli MA, Bronskill SE, Hogan DB, et al. The prevalence and health consequences of frailty in a population-based older home care cohort: a comparison of different measures. BMC Geriatr. 2016;16:133. doi: 10.1186/s12877-016-0309-z

7. Kojima G. Frailty as a predictor of future falls among community-dwelling older people: a systematic review and meta-analysis. J Am Med Dir Assoc. 2015;16:1027-1033. doi: 10.1016/j.jamda. 2015.06.018

8. Kojima G. Frailty as a predictor of fractures among community-dwelling older people: a systematic review and meta-analysis. Bone. 2016;90:116-122. doi: 10.1016/j.bone.2016.06.009

9. Kojima G. Quick and simple FRAIL scale predicts incident activities of daily living (ADL) and instrumental ADL (IADL) disabilities: a systematic review and meta-analysis. J Am Med Dir Assoc. 2018;19:1063-1068. doi: 10.1016/j.jamda.2018.07.019

10. Kojima G, Liljas AEM, Iliffe S. Frailty syndrome: implications and challenges for health care policy. Risk Manag Healthc Policy. 2019;12:23-30. doi: 10.2147/RMHP.S168750

11. Roe L, Normand C, Wren MA, et al. The impact of frailty on healthcare utilisation in Ireland: evidence from The Irish Longitudinal Study on Ageing. BMC Geriatr. 2017;17:203. doi: 10.1186/s12877-017-0579-0

12. Hao Q, Zhou L, Dong B, et al. The role of frailty in predicting mortality and readmission in older adults in acute care wards: a prospective study. Sci Rep. 2019;9:1207. doi: 10.1038/s41598-018-38072-7

13. Fried LP, Tangen CM, Walston J, et al; Cardiovascular Health Study Collaborative Research Group. Frailty in older adults: evidence for a phenotype. J Gerontol A Biol Sci Med Sci. 2001;56:M146-M156. doi: 10.1093/gerona/56.3.m146

14. Ryan J, Espinoza S, Ernst ME, et al. Validation of a deficit-­accumulation frailty Index in the ASPirin in Reducing Events in the ­Elderly study and its predictive capacity for disability-free survival. J Gerontol A Biol Sci Med Sci. 2022;77:19-26. doi: 10.1093/gerona/glab225

15. Kim DH, Glynn RJ, Avorn J, et al. Validation of a claims-based frailty index against physical performance and adverse health outcomes in the Health and Retirement Study. J Gerontol A Biol Sci Med Sci. 2019;74:1271-1276. doi: 10.1093/gerona/gly197

16. Kim DH, Schneeweiss S, Glynn RJ, et al. Measuring frailty in Medicare data: development and validation of a claims-based frailty index. J Gerontol A Biol Sci Med Sci. 2018;73:980-987. doi: 10.1093/gerona/glx229

17. Claims-based frailty index. Harvard Dataverse website. 2022. Accessed April 5, 2022. https://dataverse.harvard.edu/dataverse/cfi

18. Rockwood K, Song X, MacKnight C, et al. A global clinical measure of fitness and frailty in elderly people. CMAJ. 2005;173:489-95. doi: 10.1503/cmaj.050051

19. Dabigatran. GoodRx. Accessed September 26, 2022. www.goodrx.com/dabigatran

20. Rivaroxaban. GoodRx. Accessed September 26, 2022. www.goodrx.com/rivaroxaban

21. Apixaban (Eliquis). GoodRx. Accessed September 26, 2022. www.goodrx.com/eliquis

Article PDF
Author and Disclosure Information

Dwight David Eisenhower Army Medical Center, Fort Gordon, GA

DEPUTY EDITOR
Rebecca Mullen, MD, MPH

University of Colorado Family Medicine Residency, Denver

The views expressed in this PURL are those of the author(s) and do not reflect the official policy of the Department of the Army, the Department of Defense, or the US government.

Issue
The Journal of Family Practice - 71(9)
Publications
Topics
Page Number
407-409
Sections
Files
Files
Author and Disclosure Information

Dwight David Eisenhower Army Medical Center, Fort Gordon, GA

DEPUTY EDITOR
Rebecca Mullen, MD, MPH

University of Colorado Family Medicine Residency, Denver

The views expressed in this PURL are those of the author(s) and do not reflect the official policy of the Department of the Army, the Department of Defense, or the US government.

Author and Disclosure Information

Dwight David Eisenhower Army Medical Center, Fort Gordon, GA

DEPUTY EDITOR
Rebecca Mullen, MD, MPH

University of Colorado Family Medicine Residency, Denver

The views expressed in this PURL are those of the author(s) and do not reflect the official policy of the Department of the Army, the Department of Defense, or the US government.

Article PDF
Article PDF

ILLUSTRATIVE CASE

A frail 76-year-old woman with a history of hypertension and hyperlipidemia presents for evaluation of palpitations. An in-office electrocardiogram reveals that the patient is in AF. Her CHA2DS2-VASc score is 4 and her HAS-BLED score is 2.2,3 Using shared decision making, you decide to start medications for her AF. You plan to initiate a beta-blocker for rate control and must now decide on anticoagulation. Which oral anticoagulant would you prescribe for this patient’s AF, given her frail status?

Frailty is defined as a state of vulnerability with a decreased ability to recover from an acute stressful event.4 The prevalence of frailty varies by the measurements used and the population studied. A 2021 meta-analysis found that frailty prevalence ranges from 12% to 24% worldwide in patients older than 50 years5 and may increase to > 30% among those ages 85 years and older.6 Frailty increases rates of AEs such as falls7 and fracture,8 leading to disability,9 decreased quality of life,10 increased utilization of health care,11 and increased mortality.12 A number of validated approaches are available to screen for and measure frailty.13-18

Given the association with negative health outcomes and high health care utilization, frailty is an important clinical factor for physicians to consider when treating elderly patients. Frailty assessment may allow for more tailored treatment choices for patients, with a potential reduction in complications. Although CHA2DS2-VASc and HAS-BLED scores assist in the decision-making process of whether to start anticoagulation,these tools do not take frailty into consideration or guide anticoagulant choice.2,3 The purpose of this study was to analyze how levels of frailty affect the association of 3 different direct oral anticoagulants (DOACs) vs warfarin with various AEs (death, stroke, or major bleeding).

STUDY SUMMARY

This DOAC rose above the others

This retrospective cohort study compared the safety of 3 DOACs—dabigatran, rivaroxaban, and apixaban—vs warfarin in Medicare beneficiaries with AF, using 1:1 propensity score (PS)–matched analysis. Eligible patients were ages 65 years or older, with a filled prescription for a DOAC or warfarin, no prior oral anticoagulant exposure in the previous 183 days, a diagnostic code of AF, and continuous enrollment in Medicare Parts A, B, and D only. Patients were excluded if they had missing demographic data, received hospice care, resided in a nursing facility at drug initiation, had another indication for anticoagulation, or had a contraindication to either a DOAC or warfarin.

Frailty was measured using a claims-based frailty index (CFI), which applies health care utilization data to estimate a frailty index, with cut points for nonfrailty, prefrailty, and frailty. The CFI score has 93 claims-based variables, including wheelchairs and durable medical equipment, open wounds, diseases such as chronic obstructive pulmonary disease and ischemic heart disease, and transportation services.15-17 In this study, nonfrailty was defined as a CFI < 0.15, prefrailty as a CFI of 0.15 to 0.24, and frailty as a CFI ≥ 0.25.

Among older patients treated with anticoagulation for atrial fibrillation, apixaban had the lowest adverse event rate vs warfarin among frail patients, compared with dabigatran and rivaroxaban.

The primary outcome—a composite endpoint of death, ischemic stroke, or major bleeding—was measured for each of the DOAC–warfarin cohorts in the overall population and stratified by frailty classification. Patients were followed until the occurrence of a study outcome, Medicare disenrollment, the end of the study period, discontinuation of the index drug (defined as > 5 days), change to a different anticoagulant, admission to a nursing facility, enrollment in hospice, initiation of dialysis, or kidney transplant. The authors conducted a PS-matched analysis to reduce any imbalances in clinical characteristics between the DOAC- and warfarin-­treated groups, as well as a sensitivity analysis to assess the strength of the data findings using different assumptions.

The authors created 3 DOAC–warfarin cohorts: dabigatran (n = 81,863) vs warfarin (n = 256,722), rivaroxaban (n = 185,011) vs warfarin (n = 228,028), and apixaban (n = 222,478) vs warfarin (n = 206,031). After PS matching, the mean age in all cohorts was 76 to 77 years, about 50% were female, and 91% were White. The mean HAS-BLED score was 2 and the mean CHA2DS2-VASc score was 4. The mean CFI was 0.19 to 0.20, defined as prefrail. Patients classified as frail were older, more likely to be female, and more likely to have greater comorbidities, higher scores on CHA2DS2-VASc and HAS-BLED, and higher health care utilization.

Continue to: In the dabigatran-warfarin...

 

 

In the dabigatran–warfarin cohort (median follow-up, 72 days), the event rate of the composite endpoint per 1000 person-years (PY) was 63.5 for dabigatran and 65.6 for warfarin (hazard ratio [HR] = 0.98; 95% CI, 0.92 to 1.05; rate difference [RD] per 1000 PY = –2.2; 95% CI, –6.5 to 2.1). A lower rate of the composite endpoint was associated with dabigatran than warfarin for the nonfrail subgroup but not the prefrail or frail groups.

In the rivaroxaban–warfarin cohort (median follow-up, 82 days), the composite endpoint rate per 1000 PY was 77.8 for rivaroxaban and 83.7 for warfarin (HR = 0.98; 95% CI, 0.94 to 1.02; RD per 1000 PY = –5.9; 95% CI, –9.4 to –2.4). When stratifying by frailty category, both dabigatran and rivaroxaban were associated with a lower composite endpoint rate than warfarin for the nonfrail population only (HR = 0.81; 95% CI, 0.68 to 0.97, and HR = 0.88; 95% CI, 0.77 to 0.99, respectively).

In the apixaban–warfarin cohort (median follow-up, 84 days), the rate of the composite endpoint per 1000 PY was 60.1 for apixaban and 92.3 for warfarin (HR = 0.68; 95% CI, 0.65 to 0.72; RD per 1000 PY = –32.2; 95% CI, –36.1 to –28.3). The beneficial association for apixaban was present in all frailty categories, with an HR of 0.61 (95% CI, 0.52 to 0.71) for nonfrail patients, 0.66 (95% CI, 0.61 to 0.70) for prefrail patients, and 0.73 (95% CI, 0.67 to 0.80) for frail patients. Apixaban was the only DOAC with a relative reduction in the hazard of death, ischemic stroke, or major bleeding among all frailty groups.

WHAT’S NEW

Only apixaban had lower AE rates vs warfarin across frailty levels

Three DOACs (dabigatran, rivaroxaban, and apixaban) reduced the risk of death, ischemic stroke, or major bleeding compared with warfarin in older adults with AF, but only apixaban was associated with a relative reduction of these adverse outcomes in patients of all frailty classifications.

CAVEATS

Important data but RCTs are needed

The power of this observational study is considerable. However, it remains a retrospective observational study. The authors attempted to account for these limitations and potential confounders by performing a PS-matched analysis and sensitivity analysis; however, these findings should be confirmed with randomized controlled trials.

Continue to: Additionally, the study...

 

 

Additionally, the study collected data on each of the DOAC–warfarin cohorts for < 90 days. Trials to address long-term outcomes are warranted.

Finally, there was no control group in comparison with anticoagulation. It is possible that choosing not to use an anticoagulant is the best choice for frail elderly patients.

CHALLENGES TO IMPLEMENTATION

Doctors need a practical frailty scale, patients need an affordable Rx

Frailty is not often considered a measurable trait. The approach used in the study to determine the CFI is not a practical clinical tool. Studies comparing a frailty calculation software application or an easily implementable survey may help bring this clinically impactful information to the hands of primary care physicians. The Clinical Frailty Scale—a brief, 7-point scale based on the physician’s clinical impression of the patient—has been found to correlate with other established frailty measures18 and might be an option for busy clinicians. However, the current study did not utilize this measurement, and the validity of its use by primary care physicians in the outpatient setting requires further study.

Cost may be a barrier for patients younger than 65 years or for those older than 65 years who do not qualify for Medicare or do not have Medicare Part D.

In addition, cost may be a barrier for patients younger than 65 years or for those older than 65 years who do not qualify for Medicare or do not have Medicare Part D. The average monthly cost of the DOACs ranges from $560 for dabigatran19 to $600 for rivaroxaban20 and $623 for apixaban.21 As always, the choice of anticoagulant therapy is a clinical judgment and a joint decision of the patient and physician.

ILLUSTRATIVE CASE

A frail 76-year-old woman with a history of hypertension and hyperlipidemia presents for evaluation of palpitations. An in-office electrocardiogram reveals that the patient is in AF. Her CHA2DS2-VASc score is 4 and her HAS-BLED score is 2.2,3 Using shared decision making, you decide to start medications for her AF. You plan to initiate a beta-blocker for rate control and must now decide on anticoagulation. Which oral anticoagulant would you prescribe for this patient’s AF, given her frail status?

Frailty is defined as a state of vulnerability with a decreased ability to recover from an acute stressful event.4 The prevalence of frailty varies by the measurements used and the population studied. A 2021 meta-analysis found that frailty prevalence ranges from 12% to 24% worldwide in patients older than 50 years5 and may increase to > 30% among those ages 85 years and older.6 Frailty increases rates of AEs such as falls7 and fracture,8 leading to disability,9 decreased quality of life,10 increased utilization of health care,11 and increased mortality.12 A number of validated approaches are available to screen for and measure frailty.13-18

Given the association with negative health outcomes and high health care utilization, frailty is an important clinical factor for physicians to consider when treating elderly patients. Frailty assessment may allow for more tailored treatment choices for patients, with a potential reduction in complications. Although CHA2DS2-VASc and HAS-BLED scores assist in the decision-making process of whether to start anticoagulation,these tools do not take frailty into consideration or guide anticoagulant choice.2,3 The purpose of this study was to analyze how levels of frailty affect the association of 3 different direct oral anticoagulants (DOACs) vs warfarin with various AEs (death, stroke, or major bleeding).

STUDY SUMMARY

This DOAC rose above the others

This retrospective cohort study compared the safety of 3 DOACs—dabigatran, rivaroxaban, and apixaban—vs warfarin in Medicare beneficiaries with AF, using 1:1 propensity score (PS)–matched analysis. Eligible patients were ages 65 years or older, with a filled prescription for a DOAC or warfarin, no prior oral anticoagulant exposure in the previous 183 days, a diagnostic code of AF, and continuous enrollment in Medicare Parts A, B, and D only. Patients were excluded if they had missing demographic data, received hospice care, resided in a nursing facility at drug initiation, had another indication for anticoagulation, or had a contraindication to either a DOAC or warfarin.

Frailty was measured using a claims-based frailty index (CFI), which applies health care utilization data to estimate a frailty index, with cut points for nonfrailty, prefrailty, and frailty. The CFI score has 93 claims-based variables, including wheelchairs and durable medical equipment, open wounds, diseases such as chronic obstructive pulmonary disease and ischemic heart disease, and transportation services.15-17 In this study, nonfrailty was defined as a CFI < 0.15, prefrailty as a CFI of 0.15 to 0.24, and frailty as a CFI ≥ 0.25.

Among older patients treated with anticoagulation for atrial fibrillation, apixaban had the lowest adverse event rate vs warfarin among frail patients, compared with dabigatran and rivaroxaban.

The primary outcome—a composite endpoint of death, ischemic stroke, or major bleeding—was measured for each of the DOAC–warfarin cohorts in the overall population and stratified by frailty classification. Patients were followed until the occurrence of a study outcome, Medicare disenrollment, the end of the study period, discontinuation of the index drug (defined as > 5 days), change to a different anticoagulant, admission to a nursing facility, enrollment in hospice, initiation of dialysis, or kidney transplant. The authors conducted a PS-matched analysis to reduce any imbalances in clinical characteristics between the DOAC- and warfarin-­treated groups, as well as a sensitivity analysis to assess the strength of the data findings using different assumptions.

The authors created 3 DOAC–warfarin cohorts: dabigatran (n = 81,863) vs warfarin (n = 256,722), rivaroxaban (n = 185,011) vs warfarin (n = 228,028), and apixaban (n = 222,478) vs warfarin (n = 206,031). After PS matching, the mean age in all cohorts was 76 to 77 years, about 50% were female, and 91% were White. The mean HAS-BLED score was 2 and the mean CHA2DS2-VASc score was 4. The mean CFI was 0.19 to 0.20, defined as prefrail. Patients classified as frail were older, more likely to be female, and more likely to have greater comorbidities, higher scores on CHA2DS2-VASc and HAS-BLED, and higher health care utilization.

Continue to: In the dabigatran-warfarin...

 

 

In the dabigatran–warfarin cohort (median follow-up, 72 days), the event rate of the composite endpoint per 1000 person-years (PY) was 63.5 for dabigatran and 65.6 for warfarin (hazard ratio [HR] = 0.98; 95% CI, 0.92 to 1.05; rate difference [RD] per 1000 PY = –2.2; 95% CI, –6.5 to 2.1). A lower rate of the composite endpoint was associated with dabigatran than warfarin for the nonfrail subgroup but not the prefrail or frail groups.

In the rivaroxaban–warfarin cohort (median follow-up, 82 days), the composite endpoint rate per 1000 PY was 77.8 for rivaroxaban and 83.7 for warfarin (HR = 0.98; 95% CI, 0.94 to 1.02; RD per 1000 PY = –5.9; 95% CI, –9.4 to –2.4). When stratifying by frailty category, both dabigatran and rivaroxaban were associated with a lower composite endpoint rate than warfarin for the nonfrail population only (HR = 0.81; 95% CI, 0.68 to 0.97, and HR = 0.88; 95% CI, 0.77 to 0.99, respectively).

In the apixaban–warfarin cohort (median follow-up, 84 days), the rate of the composite endpoint per 1000 PY was 60.1 for apixaban and 92.3 for warfarin (HR = 0.68; 95% CI, 0.65 to 0.72; RD per 1000 PY = –32.2; 95% CI, –36.1 to –28.3). The beneficial association for apixaban was present in all frailty categories, with an HR of 0.61 (95% CI, 0.52 to 0.71) for nonfrail patients, 0.66 (95% CI, 0.61 to 0.70) for prefrail patients, and 0.73 (95% CI, 0.67 to 0.80) for frail patients. Apixaban was the only DOAC with a relative reduction in the hazard of death, ischemic stroke, or major bleeding among all frailty groups.

WHAT’S NEW

Only apixaban had lower AE rates vs warfarin across frailty levels

Three DOACs (dabigatran, rivaroxaban, and apixaban) reduced the risk of death, ischemic stroke, or major bleeding compared with warfarin in older adults with AF, but only apixaban was associated with a relative reduction of these adverse outcomes in patients of all frailty classifications.

CAVEATS

Important data but RCTs are needed

The power of this observational study is considerable. However, it remains a retrospective observational study. The authors attempted to account for these limitations and potential confounders by performing a PS-matched analysis and sensitivity analysis; however, these findings should be confirmed with randomized controlled trials.

Continue to: Additionally, the study...

 

 

Additionally, the study collected data on each of the DOAC–warfarin cohorts for < 90 days. Trials to address long-term outcomes are warranted.

Finally, there was no control group in comparison with anticoagulation. It is possible that choosing not to use an anticoagulant is the best choice for frail elderly patients.

CHALLENGES TO IMPLEMENTATION

Doctors need a practical frailty scale, patients need an affordable Rx

Frailty is not often considered a measurable trait. The approach used in the study to determine the CFI is not a practical clinical tool. Studies comparing a frailty calculation software application or an easily implementable survey may help bring this clinically impactful information to the hands of primary care physicians. The Clinical Frailty Scale—a brief, 7-point scale based on the physician’s clinical impression of the patient—has been found to correlate with other established frailty measures18 and might be an option for busy clinicians. However, the current study did not utilize this measurement, and the validity of its use by primary care physicians in the outpatient setting requires further study.

Cost may be a barrier for patients younger than 65 years or for those older than 65 years who do not qualify for Medicare or do not have Medicare Part D.

In addition, cost may be a barrier for patients younger than 65 years or for those older than 65 years who do not qualify for Medicare or do not have Medicare Part D. The average monthly cost of the DOACs ranges from $560 for dabigatran19 to $600 for rivaroxaban20 and $623 for apixaban.21 As always, the choice of anticoagulant therapy is a clinical judgment and a joint decision of the patient and physician.

References

1. Kim DH, Pawar A, Gagne JJ, et al. Frailty and clinical outcomes of direct oral anticoagulants versus warfarin in older adults with atrial fibrillation: a cohort study. Ann Intern Med. 2021;174:1214-1223. doi: 10.7326/M20-7141

2. Zhu W, He W, Guo L, et al. The HAS-BLED score for predicting major bleeding risk in anticoagulated patients with atrial fibrillation: a systematic review and meta-analysis. Clin Cardiol. 2015;38:555-561. doi: 10.1002/clc.22435

3. Olesen JB, Lip GYH, Hansen ML, et al. Validation of risk stratification schemes for predicting stroke and thromboembolism in patients with atrial fibrillation: nationwide cohort study. BMJ. 2011;342:d124. doi: 10.1136/bmj.d124

4. Xue QL. The frailty syndrome: definition and natural history. Clin Geriatr Med. 2011;27:1-15. doi: 10.1016/j.cger.2010.08.009

5. O’Caoimh R, Sezgin D, O’Donovan MR, et al. Prevalence of frailty in 62 countries across the world: a systematic review and meta-analysis of population-level studies. Age Ageing. 2021;50:96-104. doi: 10.1093/ageing/afaa219

6. Campitelli MA, Bronskill SE, Hogan DB, et al. The prevalence and health consequences of frailty in a population-based older home care cohort: a comparison of different measures. BMC Geriatr. 2016;16:133. doi: 10.1186/s12877-016-0309-z

7. Kojima G. Frailty as a predictor of future falls among community-dwelling older people: a systematic review and meta-analysis. J Am Med Dir Assoc. 2015;16:1027-1033. doi: 10.1016/j.jamda. 2015.06.018

8. Kojima G. Frailty as a predictor of fractures among community-dwelling older people: a systematic review and meta-analysis. Bone. 2016;90:116-122. doi: 10.1016/j.bone.2016.06.009

9. Kojima G. Quick and simple FRAIL scale predicts incident activities of daily living (ADL) and instrumental ADL (IADL) disabilities: a systematic review and meta-analysis. J Am Med Dir Assoc. 2018;19:1063-1068. doi: 10.1016/j.jamda.2018.07.019

10. Kojima G, Liljas AEM, Iliffe S. Frailty syndrome: implications and challenges for health care policy. Risk Manag Healthc Policy. 2019;12:23-30. doi: 10.2147/RMHP.S168750

11. Roe L, Normand C, Wren MA, et al. The impact of frailty on healthcare utilisation in Ireland: evidence from The Irish Longitudinal Study on Ageing. BMC Geriatr. 2017;17:203. doi: 10.1186/s12877-017-0579-0

12. Hao Q, Zhou L, Dong B, et al. The role of frailty in predicting mortality and readmission in older adults in acute care wards: a prospective study. Sci Rep. 2019;9:1207. doi: 10.1038/s41598-018-38072-7

13. Fried LP, Tangen CM, Walston J, et al; Cardiovascular Health Study Collaborative Research Group. Frailty in older adults: evidence for a phenotype. J Gerontol A Biol Sci Med Sci. 2001;56:M146-M156. doi: 10.1093/gerona/56.3.m146

14. Ryan J, Espinoza S, Ernst ME, et al. Validation of a deficit-­accumulation frailty Index in the ASPirin in Reducing Events in the ­Elderly study and its predictive capacity for disability-free survival. J Gerontol A Biol Sci Med Sci. 2022;77:19-26. doi: 10.1093/gerona/glab225

15. Kim DH, Glynn RJ, Avorn J, et al. Validation of a claims-based frailty index against physical performance and adverse health outcomes in the Health and Retirement Study. J Gerontol A Biol Sci Med Sci. 2019;74:1271-1276. doi: 10.1093/gerona/gly197

16. Kim DH, Schneeweiss S, Glynn RJ, et al. Measuring frailty in Medicare data: development and validation of a claims-based frailty index. J Gerontol A Biol Sci Med Sci. 2018;73:980-987. doi: 10.1093/gerona/glx229

17. Claims-based frailty index. Harvard Dataverse website. 2022. Accessed April 5, 2022. https://dataverse.harvard.edu/dataverse/cfi

18. Rockwood K, Song X, MacKnight C, et al. A global clinical measure of fitness and frailty in elderly people. CMAJ. 2005;173:489-95. doi: 10.1503/cmaj.050051

19. Dabigatran. GoodRx. Accessed September 26, 2022. www.goodrx.com/dabigatran

20. Rivaroxaban. GoodRx. Accessed September 26, 2022. www.goodrx.com/rivaroxaban

21. Apixaban (Eliquis). GoodRx. Accessed September 26, 2022. www.goodrx.com/eliquis

References

1. Kim DH, Pawar A, Gagne JJ, et al. Frailty and clinical outcomes of direct oral anticoagulants versus warfarin in older adults with atrial fibrillation: a cohort study. Ann Intern Med. 2021;174:1214-1223. doi: 10.7326/M20-7141

2. Zhu W, He W, Guo L, et al. The HAS-BLED score for predicting major bleeding risk in anticoagulated patients with atrial fibrillation: a systematic review and meta-analysis. Clin Cardiol. 2015;38:555-561. doi: 10.1002/clc.22435

3. Olesen JB, Lip GYH, Hansen ML, et al. Validation of risk stratification schemes for predicting stroke and thromboembolism in patients with atrial fibrillation: nationwide cohort study. BMJ. 2011;342:d124. doi: 10.1136/bmj.d124

4. Xue QL. The frailty syndrome: definition and natural history. Clin Geriatr Med. 2011;27:1-15. doi: 10.1016/j.cger.2010.08.009

5. O’Caoimh R, Sezgin D, O’Donovan MR, et al. Prevalence of frailty in 62 countries across the world: a systematic review and meta-analysis of population-level studies. Age Ageing. 2021;50:96-104. doi: 10.1093/ageing/afaa219

6. Campitelli MA, Bronskill SE, Hogan DB, et al. The prevalence and health consequences of frailty in a population-based older home care cohort: a comparison of different measures. BMC Geriatr. 2016;16:133. doi: 10.1186/s12877-016-0309-z

7. Kojima G. Frailty as a predictor of future falls among community-dwelling older people: a systematic review and meta-analysis. J Am Med Dir Assoc. 2015;16:1027-1033. doi: 10.1016/j.jamda. 2015.06.018

8. Kojima G. Frailty as a predictor of fractures among community-dwelling older people: a systematic review and meta-analysis. Bone. 2016;90:116-122. doi: 10.1016/j.bone.2016.06.009

9. Kojima G. Quick and simple FRAIL scale predicts incident activities of daily living (ADL) and instrumental ADL (IADL) disabilities: a systematic review and meta-analysis. J Am Med Dir Assoc. 2018;19:1063-1068. doi: 10.1016/j.jamda.2018.07.019

10. Kojima G, Liljas AEM, Iliffe S. Frailty syndrome: implications and challenges for health care policy. Risk Manag Healthc Policy. 2019;12:23-30. doi: 10.2147/RMHP.S168750

11. Roe L, Normand C, Wren MA, et al. The impact of frailty on healthcare utilisation in Ireland: evidence from The Irish Longitudinal Study on Ageing. BMC Geriatr. 2017;17:203. doi: 10.1186/s12877-017-0579-0

12. Hao Q, Zhou L, Dong B, et al. The role of frailty in predicting mortality and readmission in older adults in acute care wards: a prospective study. Sci Rep. 2019;9:1207. doi: 10.1038/s41598-018-38072-7

13. Fried LP, Tangen CM, Walston J, et al; Cardiovascular Health Study Collaborative Research Group. Frailty in older adults: evidence for a phenotype. J Gerontol A Biol Sci Med Sci. 2001;56:M146-M156. doi: 10.1093/gerona/56.3.m146

14. Ryan J, Espinoza S, Ernst ME, et al. Validation of a deficit-­accumulation frailty Index in the ASPirin in Reducing Events in the ­Elderly study and its predictive capacity for disability-free survival. J Gerontol A Biol Sci Med Sci. 2022;77:19-26. doi: 10.1093/gerona/glab225

15. Kim DH, Glynn RJ, Avorn J, et al. Validation of a claims-based frailty index against physical performance and adverse health outcomes in the Health and Retirement Study. J Gerontol A Biol Sci Med Sci. 2019;74:1271-1276. doi: 10.1093/gerona/gly197

16. Kim DH, Schneeweiss S, Glynn RJ, et al. Measuring frailty in Medicare data: development and validation of a claims-based frailty index. J Gerontol A Biol Sci Med Sci. 2018;73:980-987. doi: 10.1093/gerona/glx229

17. Claims-based frailty index. Harvard Dataverse website. 2022. Accessed April 5, 2022. https://dataverse.harvard.edu/dataverse/cfi

18. Rockwood K, Song X, MacKnight C, et al. A global clinical measure of fitness and frailty in elderly people. CMAJ. 2005;173:489-95. doi: 10.1503/cmaj.050051

19. Dabigatran. GoodRx. Accessed September 26, 2022. www.goodrx.com/dabigatran

20. Rivaroxaban. GoodRx. Accessed September 26, 2022. www.goodrx.com/rivaroxaban

21. Apixaban (Eliquis). GoodRx. Accessed September 26, 2022. www.goodrx.com/eliquis

Issue
The Journal of Family Practice - 71(9)
Issue
The Journal of Family Practice - 71(9)
Page Number
407-409
Page Number
407-409
Publications
Publications
Topics
Article Type
Display Headline
Which anticoagulant is safest for frail elderly patients with nonvalvular A-fib?
Display Headline
Which anticoagulant is safest for frail elderly patients with nonvalvular A-fib?
Sections
PURLs Copyright
Copyright © 2022. The Family Physicians Inquiries Network. All rights reserved.
Inside the Article

PRACTICE CHANGER

Consider apixaban, which demonstrated a lower adverse event (AE) rate than warfarin regardless of frailty status, for anticoagulation treatment of older patients with nonvalvular atrial fibrillation (AF); by comparison, AE rates for dabigatran and rivaroxaban were lower vs warfarin only among nonfrail individuals.

STRENGTH OF RECOMMENDATION

C: Based on a retrospective observational cohort study.1

Kim DH, Pawar A, Gagne JJ, et al. Frailty and clinical outcomes of direct oral anticoagulants versus warfarin in older adults with atrial fibrillation: a cohort study. Ann Intern Med. 2021;174:1214-1223. doi: 10.7326/M20-7141

Disallow All Ads
Content Gating
No Gating (article Unlocked/Free)
Alternative CME
Disqus Comments
Default
Use ProPublica
Hide sidebar & use full width
render the right sidebar.
Conference Recap Checkbox
Not Conference Recap
Clinical Edge
Display the Slideshow in this Article
Medscape Article
Display survey writer
Reuters content
Disable Inline Native ads
WebMD Article
Article PDF Media
Media Files

A “no-biopsy” approach to diagnosing celiac disease

Article Type
Changed
Thu, 10/13/2022 - 13:13
Display Headline
A “no-biopsy” approach to diagnosing celiac disease

ILLUSTRATIVE CASE

A 43-year-old woman presents to the clinic with diffuse, intermittent abdominal discomfort, bloating, and diarrhea that has slowly but steadily worsened over the past few years to now-daily symptoms. She states her overall health is otherwise good. Her review of systems is pertinent only for 8 lbs of unintentional weight loss over the past year and increased fatigue. She takes no supplements or routine over-the-counter or prescription medications, except for low-dose combination oral contraceptives, and is unaware of any family history of gastrointestinal (GI) diseases. She does not drink or smoke. She is up to date with immunizations and with cervical and breast cancer screening. Her body mass index is 23, her vital signs are within normal limits, and her physical exam is normal except for mild, diffuse abdominal tenderness without any masses, organomegaly, or peritoneal signs.

Her diagnostic work-up includes a complete metabolic panel, magnesium level, complete blood count, thyroid-stimulating hormone measurement, cytomegalovirus IgG and IgM serology, and stool studies for fecal leukocytes, ova and parasites, and fecal fat, in addition to a kidney, ureter, and bladder noncontrast computed tomography scan. All diagnostic testing is negative except for slightly elevated fecal fat, thereby decreasing the likelihood of infection, thyroid disorder, electrolyte abnormalities, or malignancy as a source of her symptoms.

She says that based on her online searches, her symptoms seem consistent with CD—with which you concur. However, she is fearful of an endoscopic procedure and asks if there is any other way to diagnose CD.

CD is an immune-mediated disorder in genetically susceptible people that is triggered by dietary gluten, causing damage to the small intestine.1-6 The estimated worldwide prevalence of CD is approximately 1%, with greater prevalence in females.1-6 A strong genetic predisposition also has been noted: prevalence among first-degree relatives is 10% to 44%.2,3,6 Although CD can be diagnosed at any age, in the United States the mean age at diagnosis is in the fifth decade of life.6

The incidence of CD is on the rise due to true increases in disease incidence and prevalence, increased detection through better diagnostic tools, and increased screening of at-risk populations (eg, first-degree relatives, those with specific human leukocyte antigen variant genotypes, and those with certain chromosomal disorders, such as Down syndrome and Turner syndrome).2-6 However, despite the increasing prevalence of CD, most patients remain undiagnosed.1

The consistently strong predictive value of tTG-IgA serum testing may enable celiac disease diagnosis at a much lower cost and reduced risk vs traditional invasive procedures.

The diagnosis of CD in adults is typically made with elevated serum tTG-IgA and endomysial IgA antibodies (EMAs) on initial screening, followed by a duodenal biopsy via EGD for confirmatory testing and/or elucidation of differential diagnoses.7,8 In 2020, guidelines from the European Society for Paediatric Gastroenterology, Hepatology and Nutrition advised that the diagnosis of CD in children can be made without the need for biopsy.9 They stated that serum tTG-IgA antibodies ≥ 10 times the ULN, in conjunction with a positive serum EMA, effectively make the diagnosis without endoscopy. Although the gold standard of EGD with biopsy for diagnosing CD has its own inherent risks and can be expensive, a “no-biopsy” approach has yet to be adopted into guidelines for diagnosing CD in adults.7,8

STUDY SUMMARY

tTG-IgA titers were highly predictive of CD in 3 distinct cohorts

This 2021 hybrid prospective/retrospective study with 3 distinct cohorts aimed to assess the utility of serum tTG-IgA titers compared to traditional EGD with duodenal biopsy for the diagnosis of CD in adult participants (defined as ≥ 16 years of age). A serum tTG-IgA titer ≥ 10 times the ULN was set as the minimal cutoff value, and standardized duodenal biopsy sampling and evaluation for histologic mucosal changes consistent with Marsh 3 lesions was used as the diagnostic reference standard.

Continue to: Cohort 1 was a...

 

 

Cohort 1 was a prospective analysis of adults (N = 740) considered to have a high suspicion for CD, recruited from a single CD subspecialty clinic in the United Kingdom. Patients with a previous diagnosis of CD, those adhering to a gluten-free diet, and those with IgA deficiency were excluded. Study patients had tTG-IgA titers drawn and, within 6 weeks, underwent endoscopy with ≥ 1 biopsy from the duodenal bulb and/or the second part of the duodenum. The PPV of tTG-IgA titers ≥ 10 times the ULN in patients with biopsy-proven CD was 98.7% (95% CI, 97%-99.4%).

Cohort 2 was a retrospective analysis of adult patients (N = 532) considered to have low suspicion for CD. These patients were referred for endoscopy for generalized GI complaints in the same hospital as Cohort 1, but not the subspecialty clinic. Exclusion criteria and timing of IgA titers and endoscopy were identical to those of Cohort 1. The PPV of tTG-IgA titers ≥ 10 times the ULN in patients with biopsy-proven CD was 100%.

Cohort 3 (which included patients in 8 countries) was a retrospective analysis of the performance of multiple assays to enhance the validity of this approach in a wide range of settings. Adult patients (N = 145) with tTG-IgA serology positive for celiac who then underwent endoscopy with 4 to 6 duodenal biopsy samples were included in this analysis. Eleven distinct laboratories performed the tTG-IgA assay. The PPV of tTG-IgA titers ≥ 10 times the ULN in patients with biopsy-proven CD was 95.2% (95% CI, 84.6%-98.6%).

In total, this study included 1417 adult patients; 431 (30%) had tTG-IgA titers ≥ 10 times the ULN. Of those patients, 424 (98%) had histopathologic findings on duodenal biopsy consistent with CD.

Of note, there was no standardization as to the assays used for the tTG-IgA titers: Cohort 1 used 2 different manufacturers’ assays, Cohort 2 used 1 assay, and Cohort 3 used 5 assays. Regardless, the “≥ 10 times the ULN” calculation was based on each manufacturer’s published assay ranges. The lack of assay standardization did create variance in false-positive rates, however: Across all 3 cohorts, the false-positive rate for trusting the “≥ 10 times the ULN” threshold as the sole marker for CD in adults increased from 1% (Cohorts 1 and 2) to 5% (all 3 cohorts).

Continue to: WHAT'S NEW

 

 

WHAT’S NEW

Less invasive, less costly diagnosis of celiac disease in adults

In adults with symptoms suggestive of CD, the diagnosis can be made with a high level of certainty if a serum tTG-IgA titer is ≥ 10 times the ULN. Through informed, shared decision making in the presence of such a finding, patients may accept a serologic diagnosis and forgo an invasive EGD with biopsy and its inherent costs and risks. Indeed, if the majority of patients with CD are undiagnosed or underdiagnosed, and there exists a minimally invasive blood test that is highly cost effective in the absence of “red flags,” the overall benefit of this path could be substantial.

CAVEATS

“No biopsy” does not mean no risk/benefit discussion

While the PPVs are quite high, the negative predictive value varied greatly: 13%, 98%, and 10% for Cohorts 1, 2, and 3, respectively. Therefore, although serum tTG-IgA titers ≥ 10 times the ULN are useful for diagnosis, a negative result (serum tTG-IgA titers < 10 times the ULN) should not be used to rule out CD, and other testing should be pursued.

Additionally (although rare), patients with CD who have IgA deficiency may obtain false-negative results using the tTG-IgA ≥ 10 times the ULN diagnostic criterion.7,8

Also, both Cohorts 1 and 2 took place in general or subspecialty GI clinics (Cohort 3’s site types were not specified). However, the objective interpretation of tTG-IgA serology means it could be considered as an additional diagnostic tool for primary care physicians, as well.

If there are any potential “red flag” symptoms suggesting the possibility of a more concerning differential diagnosis, EGD evaluation should still be pursued.

Finally, if a primary care physician and their patient decide to go the “no-biopsy” route, it should be with a full discussion of the possible risks and benefits of not pursuing EGD. If there are any potential “red flag” symptoms suggesting the possibility of a more concerning differential diagnosis, EGD evaluation should still be pursued. Such symptoms might include (but not be limited to) chronic dyspepsia, dysphagia, weight loss, and unexplained anemia.7

Continue to: CHALLENGES TO IMPLEMENTATION

 

 

CHALLENGES TO IMPLEMENTATION

Diagnostic guidelines still favor EGD with biopsy for adults

The 2013 American College of Gastroenterology guidelines support the use of EGD and duodenal biopsy to diagnose CD in both low- and high-risk patients, regardless of serologic findings.7 In a 2019 Clinical Practice Update, the American Gastrointestinal Association (AGA) stated that when tTG-IgA titers are ≥ 10 times the ULN and EMAs are positive, the PPV is “virtually 100%” for CD. Yet they still state that in this scenario “EGD and duodenal biopsies may then be performed for purposes of differential diagnosis.”8 Furthermore, the AGA does not discuss informed and shared decision making with patients for the option of a “no-biopsy” diagnosis.8

Additionally, there may be challenges in finding commercial laboratories that report reference ranges with a clear ULN. Although costs for the serum tTG-IgA assay vary, they are less expensive than endoscopy with biopsy and histopathologic examination, and therefore may present less of a financial barrier.

Files
References

1. Penny HA, Raju SA, Lau MS, et al. Accuracy of a no-biopsy approach for the diagnosis of coeliac disease across different adult cohorts. Gut. 2021;70:876-883. doi: 10.1136/gutjnl-2020-320913

2. Al-Toma A, Volta U, Auricchio R, et al. European Society for the Study of Coeliac Disease (ESsCD) guideline for coeliac disease and other gluten-related disorders. United European Gastroenterol J. 2019;7:583-613. doi: 10.1177/2050640619844125

3. Caio G, Volta U, Sapone A, et al. Celiac disease: a comprehensive current review. BMC Med. 2019;17:142. doi: 10.1186/s12916-019-1380-z

4. Lebwohl B, Rubio-Tapia A. Epidemiology, presentation, and diagnosis of celiac disease. Gastroenterology. 2021;160:63-75. doi: 10.1053/j.gastro.2020.06.098

5. Lebwohl B, Sanders DS, Green PHR. Coeliac disease. Lancet. 2018;391:70-81. doi: 10.1016/S0140-6736(17)31796-8

6. Rubin JE, Crowe SE. Celiac disease. Ann Intern Med. 2020;172:ITC1-ITC16. doi: 10.7326/AITC202001070

7. Rubio-Tapia A, Hill ID, Kelly CP, et al; American College of Gastroenterology. ACG clinical guidelines: diagnosis and management of celiac disease. Am J Gastroenterol. 2013;108:656-676; quiz 677. doi: 10.1038/ajg.2013.79

8. Husby S, Murray JA, Katzka DA. AGA clinical practice update on diagnosis and monitoring of celiac disease—changing utility of serology and histologic measures: expert review. Gastroenterology. 2019;156:885-889. doi: 10.1053/j.gastro.2018.12.010

9. Husby S, Koletzko S, Korponay-Szabó I, et al. European Society Paediatric Gastroenterology, Hepatology and Nutrition guidelines for diagnosing coeliac disease 2020. J Pediatr Gastroenterol Nutr. 2020;70:141-156. doi: 10.1097/MPG.0000000000002497

Article PDF
Author and Disclosure Information

South Baldwin Regional Medical Center Family Medicine Residency Program, Foley, AL

DEPUTY EDITOR
Katherine Hale, PharmD, BCPS

Kadlec Regional Medical Center, Richland, WA

Issue
The Journal of Family Practice - 71(8)
Publications
Topics
Page Number
359-361
Sections
Files
Files
Author and Disclosure Information

South Baldwin Regional Medical Center Family Medicine Residency Program, Foley, AL

DEPUTY EDITOR
Katherine Hale, PharmD, BCPS

Kadlec Regional Medical Center, Richland, WA

Author and Disclosure Information

South Baldwin Regional Medical Center Family Medicine Residency Program, Foley, AL

DEPUTY EDITOR
Katherine Hale, PharmD, BCPS

Kadlec Regional Medical Center, Richland, WA

Article PDF
Article PDF

ILLUSTRATIVE CASE

A 43-year-old woman presents to the clinic with diffuse, intermittent abdominal discomfort, bloating, and diarrhea that has slowly but steadily worsened over the past few years to now-daily symptoms. She states her overall health is otherwise good. Her review of systems is pertinent only for 8 lbs of unintentional weight loss over the past year and increased fatigue. She takes no supplements or routine over-the-counter or prescription medications, except for low-dose combination oral contraceptives, and is unaware of any family history of gastrointestinal (GI) diseases. She does not drink or smoke. She is up to date with immunizations and with cervical and breast cancer screening. Her body mass index is 23, her vital signs are within normal limits, and her physical exam is normal except for mild, diffuse abdominal tenderness without any masses, organomegaly, or peritoneal signs.

Her diagnostic work-up includes a complete metabolic panel, magnesium level, complete blood count, thyroid-stimulating hormone measurement, cytomegalovirus IgG and IgM serology, and stool studies for fecal leukocytes, ova and parasites, and fecal fat, in addition to a kidney, ureter, and bladder noncontrast computed tomography scan. All diagnostic testing is negative except for slightly elevated fecal fat, thereby decreasing the likelihood of infection, thyroid disorder, electrolyte abnormalities, or malignancy as a source of her symptoms.

She says that based on her online searches, her symptoms seem consistent with CD—with which you concur. However, she is fearful of an endoscopic procedure and asks if there is any other way to diagnose CD.

CD is an immune-mediated disorder in genetically susceptible people that is triggered by dietary gluten, causing damage to the small intestine.1-6 The estimated worldwide prevalence of CD is approximately 1%, with greater prevalence in females.1-6 A strong genetic predisposition also has been noted: prevalence among first-degree relatives is 10% to 44%.2,3,6 Although CD can be diagnosed at any age, in the United States the mean age at diagnosis is in the fifth decade of life.6

The incidence of CD is on the rise due to true increases in disease incidence and prevalence, increased detection through better diagnostic tools, and increased screening of at-risk populations (eg, first-degree relatives, those with specific human leukocyte antigen variant genotypes, and those with certain chromosomal disorders, such as Down syndrome and Turner syndrome).2-6 However, despite the increasing prevalence of CD, most patients remain undiagnosed.1

The consistently strong predictive value of tTG-IgA serum testing may enable celiac disease diagnosis at a much lower cost and reduced risk vs traditional invasive procedures.

The diagnosis of CD in adults is typically made with elevated serum tTG-IgA and endomysial IgA antibodies (EMAs) on initial screening, followed by a duodenal biopsy via EGD for confirmatory testing and/or elucidation of differential diagnoses.7,8 In 2020, guidelines from the European Society for Paediatric Gastroenterology, Hepatology and Nutrition advised that the diagnosis of CD in children can be made without the need for biopsy.9 They stated that serum tTG-IgA antibodies ≥ 10 times the ULN, in conjunction with a positive serum EMA, effectively make the diagnosis without endoscopy. Although the gold standard of EGD with biopsy for diagnosing CD has its own inherent risks and can be expensive, a “no-biopsy” approach has yet to be adopted into guidelines for diagnosing CD in adults.7,8

STUDY SUMMARY

tTG-IgA titers were highly predictive of CD in 3 distinct cohorts

This 2021 hybrid prospective/retrospective study with 3 distinct cohorts aimed to assess the utility of serum tTG-IgA titers compared to traditional EGD with duodenal biopsy for the diagnosis of CD in adult participants (defined as ≥ 16 years of age). A serum tTG-IgA titer ≥ 10 times the ULN was set as the minimal cutoff value, and standardized duodenal biopsy sampling and evaluation for histologic mucosal changes consistent with Marsh 3 lesions was used as the diagnostic reference standard.

Continue to: Cohort 1 was a...

 

 

Cohort 1 was a prospective analysis of adults (N = 740) considered to have a high suspicion for CD, recruited from a single CD subspecialty clinic in the United Kingdom. Patients with a previous diagnosis of CD, those adhering to a gluten-free diet, and those with IgA deficiency were excluded. Study patients had tTG-IgA titers drawn and, within 6 weeks, underwent endoscopy with ≥ 1 biopsy from the duodenal bulb and/or the second part of the duodenum. The PPV of tTG-IgA titers ≥ 10 times the ULN in patients with biopsy-proven CD was 98.7% (95% CI, 97%-99.4%).

Cohort 2 was a retrospective analysis of adult patients (N = 532) considered to have low suspicion for CD. These patients were referred for endoscopy for generalized GI complaints in the same hospital as Cohort 1, but not the subspecialty clinic. Exclusion criteria and timing of IgA titers and endoscopy were identical to those of Cohort 1. The PPV of tTG-IgA titers ≥ 10 times the ULN in patients with biopsy-proven CD was 100%.

Cohort 3 (which included patients in 8 countries) was a retrospective analysis of the performance of multiple assays to enhance the validity of this approach in a wide range of settings. Adult patients (N = 145) with tTG-IgA serology positive for celiac who then underwent endoscopy with 4 to 6 duodenal biopsy samples were included in this analysis. Eleven distinct laboratories performed the tTG-IgA assay. The PPV of tTG-IgA titers ≥ 10 times the ULN in patients with biopsy-proven CD was 95.2% (95% CI, 84.6%-98.6%).

In total, this study included 1417 adult patients; 431 (30%) had tTG-IgA titers ≥ 10 times the ULN. Of those patients, 424 (98%) had histopathologic findings on duodenal biopsy consistent with CD.

Of note, there was no standardization as to the assays used for the tTG-IgA titers: Cohort 1 used 2 different manufacturers’ assays, Cohort 2 used 1 assay, and Cohort 3 used 5 assays. Regardless, the “≥ 10 times the ULN” calculation was based on each manufacturer’s published assay ranges. The lack of assay standardization did create variance in false-positive rates, however: Across all 3 cohorts, the false-positive rate for trusting the “≥ 10 times the ULN” threshold as the sole marker for CD in adults increased from 1% (Cohorts 1 and 2) to 5% (all 3 cohorts).

Continue to: WHAT'S NEW

 

 

WHAT’S NEW

Less invasive, less costly diagnosis of celiac disease in adults

In adults with symptoms suggestive of CD, the diagnosis can be made with a high level of certainty if a serum tTG-IgA titer is ≥ 10 times the ULN. Through informed, shared decision making in the presence of such a finding, patients may accept a serologic diagnosis and forgo an invasive EGD with biopsy and its inherent costs and risks. Indeed, if the majority of patients with CD are undiagnosed or underdiagnosed, and there exists a minimally invasive blood test that is highly cost effective in the absence of “red flags,” the overall benefit of this path could be substantial.

CAVEATS

“No biopsy” does not mean no risk/benefit discussion

While the PPVs are quite high, the negative predictive value varied greatly: 13%, 98%, and 10% for Cohorts 1, 2, and 3, respectively. Therefore, although serum tTG-IgA titers ≥ 10 times the ULN are useful for diagnosis, a negative result (serum tTG-IgA titers < 10 times the ULN) should not be used to rule out CD, and other testing should be pursued.

Additionally (although rare), patients with CD who have IgA deficiency may obtain false-negative results using the tTG-IgA ≥ 10 times the ULN diagnostic criterion.7,8

Also, both Cohorts 1 and 2 took place in general or subspecialty GI clinics (Cohort 3’s site types were not specified). However, the objective interpretation of tTG-IgA serology means it could be considered as an additional diagnostic tool for primary care physicians, as well.

If there are any potential “red flag” symptoms suggesting the possibility of a more concerning differential diagnosis, EGD evaluation should still be pursued.

Finally, if a primary care physician and their patient decide to go the “no-biopsy” route, it should be with a full discussion of the possible risks and benefits of not pursuing EGD. If there are any potential “red flag” symptoms suggesting the possibility of a more concerning differential diagnosis, EGD evaluation should still be pursued. Such symptoms might include (but not be limited to) chronic dyspepsia, dysphagia, weight loss, and unexplained anemia.7

Continue to: CHALLENGES TO IMPLEMENTATION

 

 

CHALLENGES TO IMPLEMENTATION

Diagnostic guidelines still favor EGD with biopsy for adults

The 2013 American College of Gastroenterology guidelines support the use of EGD and duodenal biopsy to diagnose CD in both low- and high-risk patients, regardless of serologic findings.7 In a 2019 Clinical Practice Update, the American Gastrointestinal Association (AGA) stated that when tTG-IgA titers are ≥ 10 times the ULN and EMAs are positive, the PPV is “virtually 100%” for CD. Yet they still state that in this scenario “EGD and duodenal biopsies may then be performed for purposes of differential diagnosis.”8 Furthermore, the AGA does not discuss informed and shared decision making with patients for the option of a “no-biopsy” diagnosis.8

Additionally, there may be challenges in finding commercial laboratories that report reference ranges with a clear ULN. Although costs for the serum tTG-IgA assay vary, they are less expensive than endoscopy with biopsy and histopathologic examination, and therefore may present less of a financial barrier.

ILLUSTRATIVE CASE

A 43-year-old woman presents to the clinic with diffuse, intermittent abdominal discomfort, bloating, and diarrhea that has slowly but steadily worsened over the past few years to now-daily symptoms. She states her overall health is otherwise good. Her review of systems is pertinent only for 8 lbs of unintentional weight loss over the past year and increased fatigue. She takes no supplements or routine over-the-counter or prescription medications, except for low-dose combination oral contraceptives, and is unaware of any family history of gastrointestinal (GI) diseases. She does not drink or smoke. She is up to date with immunizations and with cervical and breast cancer screening. Her body mass index is 23, her vital signs are within normal limits, and her physical exam is normal except for mild, diffuse abdominal tenderness without any masses, organomegaly, or peritoneal signs.

Her diagnostic work-up includes a complete metabolic panel, magnesium level, complete blood count, thyroid-stimulating hormone measurement, cytomegalovirus IgG and IgM serology, and stool studies for fecal leukocytes, ova and parasites, and fecal fat, in addition to a kidney, ureter, and bladder noncontrast computed tomography scan. All diagnostic testing is negative except for slightly elevated fecal fat, thereby decreasing the likelihood of infection, thyroid disorder, electrolyte abnormalities, or malignancy as a source of her symptoms.

She says that based on her online searches, her symptoms seem consistent with CD—with which you concur. However, she is fearful of an endoscopic procedure and asks if there is any other way to diagnose CD.

CD is an immune-mediated disorder in genetically susceptible people that is triggered by dietary gluten, causing damage to the small intestine.1-6 The estimated worldwide prevalence of CD is approximately 1%, with greater prevalence in females.1-6 A strong genetic predisposition also has been noted: prevalence among first-degree relatives is 10% to 44%.2,3,6 Although CD can be diagnosed at any age, in the United States the mean age at diagnosis is in the fifth decade of life.6

The incidence of CD is on the rise due to true increases in disease incidence and prevalence, increased detection through better diagnostic tools, and increased screening of at-risk populations (eg, first-degree relatives, those with specific human leukocyte antigen variant genotypes, and those with certain chromosomal disorders, such as Down syndrome and Turner syndrome).2-6 However, despite the increasing prevalence of CD, most patients remain undiagnosed.1

The consistently strong predictive value of tTG-IgA serum testing may enable celiac disease diagnosis at a much lower cost and reduced risk vs traditional invasive procedures.

The diagnosis of CD in adults is typically made with elevated serum tTG-IgA and endomysial IgA antibodies (EMAs) on initial screening, followed by a duodenal biopsy via EGD for confirmatory testing and/or elucidation of differential diagnoses.7,8 In 2020, guidelines from the European Society for Paediatric Gastroenterology, Hepatology and Nutrition advised that the diagnosis of CD in children can be made without the need for biopsy.9 They stated that serum tTG-IgA antibodies ≥ 10 times the ULN, in conjunction with a positive serum EMA, effectively make the diagnosis without endoscopy. Although the gold standard of EGD with biopsy for diagnosing CD has its own inherent risks and can be expensive, a “no-biopsy” approach has yet to be adopted into guidelines for diagnosing CD in adults.7,8

STUDY SUMMARY

tTG-IgA titers were highly predictive of CD in 3 distinct cohorts

This 2021 hybrid prospective/retrospective study with 3 distinct cohorts aimed to assess the utility of serum tTG-IgA titers compared to traditional EGD with duodenal biopsy for the diagnosis of CD in adult participants (defined as ≥ 16 years of age). A serum tTG-IgA titer ≥ 10 times the ULN was set as the minimal cutoff value, and standardized duodenal biopsy sampling and evaluation for histologic mucosal changes consistent with Marsh 3 lesions was used as the diagnostic reference standard.

Continue to: Cohort 1 was a...

 

 

Cohort 1 was a prospective analysis of adults (N = 740) considered to have a high suspicion for CD, recruited from a single CD subspecialty clinic in the United Kingdom. Patients with a previous diagnosis of CD, those adhering to a gluten-free diet, and those with IgA deficiency were excluded. Study patients had tTG-IgA titers drawn and, within 6 weeks, underwent endoscopy with ≥ 1 biopsy from the duodenal bulb and/or the second part of the duodenum. The PPV of tTG-IgA titers ≥ 10 times the ULN in patients with biopsy-proven CD was 98.7% (95% CI, 97%-99.4%).

Cohort 2 was a retrospective analysis of adult patients (N = 532) considered to have low suspicion for CD. These patients were referred for endoscopy for generalized GI complaints in the same hospital as Cohort 1, but not the subspecialty clinic. Exclusion criteria and timing of IgA titers and endoscopy were identical to those of Cohort 1. The PPV of tTG-IgA titers ≥ 10 times the ULN in patients with biopsy-proven CD was 100%.

Cohort 3 (which included patients in 8 countries) was a retrospective analysis of the performance of multiple assays to enhance the validity of this approach in a wide range of settings. Adult patients (N = 145) with tTG-IgA serology positive for celiac who then underwent endoscopy with 4 to 6 duodenal biopsy samples were included in this analysis. Eleven distinct laboratories performed the tTG-IgA assay. The PPV of tTG-IgA titers ≥ 10 times the ULN in patients with biopsy-proven CD was 95.2% (95% CI, 84.6%-98.6%).

In total, this study included 1417 adult patients; 431 (30%) had tTG-IgA titers ≥ 10 times the ULN. Of those patients, 424 (98%) had histopathologic findings on duodenal biopsy consistent with CD.

Of note, there was no standardization as to the assays used for the tTG-IgA titers: Cohort 1 used 2 different manufacturers’ assays, Cohort 2 used 1 assay, and Cohort 3 used 5 assays. Regardless, the “≥ 10 times the ULN” calculation was based on each manufacturer’s published assay ranges. The lack of assay standardization did create variance in false-positive rates, however: Across all 3 cohorts, the false-positive rate for trusting the “≥ 10 times the ULN” threshold as the sole marker for CD in adults increased from 1% (Cohorts 1 and 2) to 5% (all 3 cohorts).

Continue to: WHAT'S NEW

 

 

WHAT’S NEW

Less invasive, less costly diagnosis of celiac disease in adults

In adults with symptoms suggestive of CD, the diagnosis can be made with a high level of certainty if a serum tTG-IgA titer is ≥ 10 times the ULN. Through informed, shared decision making in the presence of such a finding, patients may accept a serologic diagnosis and forgo an invasive EGD with biopsy and its inherent costs and risks. Indeed, if the majority of patients with CD are undiagnosed or underdiagnosed, and there exists a minimally invasive blood test that is highly cost effective in the absence of “red flags,” the overall benefit of this path could be substantial.

CAVEATS

“No biopsy” does not mean no risk/benefit discussion

While the PPVs are quite high, the negative predictive value varied greatly: 13%, 98%, and 10% for Cohorts 1, 2, and 3, respectively. Therefore, although serum tTG-IgA titers ≥ 10 times the ULN are useful for diagnosis, a negative result (serum tTG-IgA titers < 10 times the ULN) should not be used to rule out CD, and other testing should be pursued.

Additionally (although rare), patients with CD who have IgA deficiency may obtain false-negative results using the tTG-IgA ≥ 10 times the ULN diagnostic criterion.7,8

Also, both Cohorts 1 and 2 took place in general or subspecialty GI clinics (Cohort 3’s site types were not specified). However, the objective interpretation of tTG-IgA serology means it could be considered as an additional diagnostic tool for primary care physicians, as well.

If there are any potential “red flag” symptoms suggesting the possibility of a more concerning differential diagnosis, EGD evaluation should still be pursued.

Finally, if a primary care physician and their patient decide to go the “no-biopsy” route, it should be with a full discussion of the possible risks and benefits of not pursuing EGD. If there are any potential “red flag” symptoms suggesting the possibility of a more concerning differential diagnosis, EGD evaluation should still be pursued. Such symptoms might include (but not be limited to) chronic dyspepsia, dysphagia, weight loss, and unexplained anemia.7

Continue to: CHALLENGES TO IMPLEMENTATION

 

 

CHALLENGES TO IMPLEMENTATION

Diagnostic guidelines still favor EGD with biopsy for adults

The 2013 American College of Gastroenterology guidelines support the use of EGD and duodenal biopsy to diagnose CD in both low- and high-risk patients, regardless of serologic findings.7 In a 2019 Clinical Practice Update, the American Gastrointestinal Association (AGA) stated that when tTG-IgA titers are ≥ 10 times the ULN and EMAs are positive, the PPV is “virtually 100%” for CD. Yet they still state that in this scenario “EGD and duodenal biopsies may then be performed for purposes of differential diagnosis.”8 Furthermore, the AGA does not discuss informed and shared decision making with patients for the option of a “no-biopsy” diagnosis.8

Additionally, there may be challenges in finding commercial laboratories that report reference ranges with a clear ULN. Although costs for the serum tTG-IgA assay vary, they are less expensive than endoscopy with biopsy and histopathologic examination, and therefore may present less of a financial barrier.

References

1. Penny HA, Raju SA, Lau MS, et al. Accuracy of a no-biopsy approach for the diagnosis of coeliac disease across different adult cohorts. Gut. 2021;70:876-883. doi: 10.1136/gutjnl-2020-320913

2. Al-Toma A, Volta U, Auricchio R, et al. European Society for the Study of Coeliac Disease (ESsCD) guideline for coeliac disease and other gluten-related disorders. United European Gastroenterol J. 2019;7:583-613. doi: 10.1177/2050640619844125

3. Caio G, Volta U, Sapone A, et al. Celiac disease: a comprehensive current review. BMC Med. 2019;17:142. doi: 10.1186/s12916-019-1380-z

4. Lebwohl B, Rubio-Tapia A. Epidemiology, presentation, and diagnosis of celiac disease. Gastroenterology. 2021;160:63-75. doi: 10.1053/j.gastro.2020.06.098

5. Lebwohl B, Sanders DS, Green PHR. Coeliac disease. Lancet. 2018;391:70-81. doi: 10.1016/S0140-6736(17)31796-8

6. Rubin JE, Crowe SE. Celiac disease. Ann Intern Med. 2020;172:ITC1-ITC16. doi: 10.7326/AITC202001070

7. Rubio-Tapia A, Hill ID, Kelly CP, et al; American College of Gastroenterology. ACG clinical guidelines: diagnosis and management of celiac disease. Am J Gastroenterol. 2013;108:656-676; quiz 677. doi: 10.1038/ajg.2013.79

8. Husby S, Murray JA, Katzka DA. AGA clinical practice update on diagnosis and monitoring of celiac disease—changing utility of serology and histologic measures: expert review. Gastroenterology. 2019;156:885-889. doi: 10.1053/j.gastro.2018.12.010

9. Husby S, Koletzko S, Korponay-Szabó I, et al. European Society Paediatric Gastroenterology, Hepatology and Nutrition guidelines for diagnosing coeliac disease 2020. J Pediatr Gastroenterol Nutr. 2020;70:141-156. doi: 10.1097/MPG.0000000000002497

References

1. Penny HA, Raju SA, Lau MS, et al. Accuracy of a no-biopsy approach for the diagnosis of coeliac disease across different adult cohorts. Gut. 2021;70:876-883. doi: 10.1136/gutjnl-2020-320913

2. Al-Toma A, Volta U, Auricchio R, et al. European Society for the Study of Coeliac Disease (ESsCD) guideline for coeliac disease and other gluten-related disorders. United European Gastroenterol J. 2019;7:583-613. doi: 10.1177/2050640619844125

3. Caio G, Volta U, Sapone A, et al. Celiac disease: a comprehensive current review. BMC Med. 2019;17:142. doi: 10.1186/s12916-019-1380-z

4. Lebwohl B, Rubio-Tapia A. Epidemiology, presentation, and diagnosis of celiac disease. Gastroenterology. 2021;160:63-75. doi: 10.1053/j.gastro.2020.06.098

5. Lebwohl B, Sanders DS, Green PHR. Coeliac disease. Lancet. 2018;391:70-81. doi: 10.1016/S0140-6736(17)31796-8

6. Rubin JE, Crowe SE. Celiac disease. Ann Intern Med. 2020;172:ITC1-ITC16. doi: 10.7326/AITC202001070

7. Rubio-Tapia A, Hill ID, Kelly CP, et al; American College of Gastroenterology. ACG clinical guidelines: diagnosis and management of celiac disease. Am J Gastroenterol. 2013;108:656-676; quiz 677. doi: 10.1038/ajg.2013.79

8. Husby S, Murray JA, Katzka DA. AGA clinical practice update on diagnosis and monitoring of celiac disease—changing utility of serology and histologic measures: expert review. Gastroenterology. 2019;156:885-889. doi: 10.1053/j.gastro.2018.12.010

9. Husby S, Koletzko S, Korponay-Szabó I, et al. European Society Paediatric Gastroenterology, Hepatology and Nutrition guidelines for diagnosing coeliac disease 2020. J Pediatr Gastroenterol Nutr. 2020;70:141-156. doi: 10.1097/MPG.0000000000002497

Issue
The Journal of Family Practice - 71(8)
Issue
The Journal of Family Practice - 71(8)
Page Number
359-361
Page Number
359-361
Publications
Publications
Topics
Article Type
Display Headline
A “no-biopsy” approach to diagnosing celiac disease
Display Headline
A “no-biopsy” approach to diagnosing celiac disease
Sections
PURLs Copyright
Copyright © 2022. The Family Physicians Inquiries Network. All rights reserved.
Inside the Article

PRACTICE CHANGER

Consider a “no-biopsy” approach by evaluating serum immunoglobulin (Ig) A anti-tissue transglutaminase (tTG-IgA) antibody titers in adult patients who present with symptoms concerning for celiac disease (CD). An increase of ≥ 10 times the upper limit of normal (ULN) for tTG-IgA has a positive predictive value (PPV) of ≥ 95% for diagnosing CD when compared with esophagogastroduodenoscopy (EGD) with duodenal biopsy—the current gold standard.

STRENGTH OF RECOMMENDATION

A: Consistent findings from 3 good-quality diagnostic cohorts presented in a single study.1

Penny HA, Raju SA, Lau MS, et al. Accuracy of a no-biopsy approach for the diagnosis of coeliac disease across different adult cohorts. Gut. 2021;70:876-883. doi: 10.1136/gutjnl-2020-320913

Disallow All Ads
Content Gating
No Gating (article Unlocked/Free)
Alternative CME
Disqus Comments
Default
Use ProPublica
Hide sidebar & use full width
render the right sidebar.
Conference Recap Checkbox
Not Conference Recap
Clinical Edge
Display the Slideshow in this Article
Medscape Article
Display survey writer
Reuters content
Disable Inline Native ads
WebMD Article
Article PDF Media
Media Files

Noncardiac inpatient has acute hypertension: Treat or not?

Article Type
Changed
Wed, 09/14/2022 - 16:53
Display Headline
Noncardiac inpatient has acute hypertension: Treat or not?

ILLUSTRATIVE CASE

A 48-year-old man is admitted to your family medicine service for cellulitis after failed outpatient therapy. He has presumed community-acquired methicillin-resistant Staphylococcus aureus infection of the left lower extremity and is receiving intravenous (IV) vancomycin. His BP this morning is 176/98 mm Hg, and the reading from the previous shift was 168/94 mm Hg. He is asymptomatic from this elevated BP. Based on protocol, his nurse is asking about treatment in response to the multiple elevated readings. How should you address the patient’s elevated BP, knowing that you will see him for a transition management appointment in 2 weeks?

Elevated BP is common in the adult inpatient setting. Prevalence estimates range from 25% to > 50%. Many factors can contribute to elevated BP in the acute illness setting, such as pain, anxiety, medication withdrawal, and volume status.2,3

Treatment of elevated BP in outpatients is well researched, with evidence-based guidelines for physicians. That is not the case for treatment of asymptomatic elevated BP in the inpatient setting. Most published guidance on inpatient management of acutely elevated BP recommends IV medications, such as hydralazine or labetalol, although there is limited evidence to support such recommendations. There is minimal evidence for outcomes-based benefit in treating acute elevations of inpatient BP, such as reduced myocardial injury or stroke; however, there is some evidence of adverse outcomes, such as hypotension and prolonged hospital stays.4-8

Although the possibility of intensifying antihypertensive therapy for those with known hypertension or those with presumed “new-onset” hypertension could theoretically lead to improved outcomes over the long term, there is little evidence to support this presumption. Rather, there is evidence that intensification of antihypertensive therapy at discharge is linked to short-term harms. This was demonstrated in a propensity-matched veteran cohort that included 4056 hospitalized older adults with hypertension (mean age, 77 years; 3961 men), equally split between those who received antihypertensive intensification at hospital discharge and those who did not. Within 30 days, patients receiving intensification had a higher risk of readmission (number needed to harm [NNH] = 27) and serious adverse events (NNH = 63).9

The current study aimed to put all these pieces together by quantifying the prevalence of hypertension in hospitalized patients, characterizing clinician response to patients’ acutely elevated BP, and comparing both short- and long-term outcomes in patients treated for acute BP elevations while hospitalized vs those who were not. The study also assessed the potential effects of antihypertensive intensification at discharge.

STUDY SUMMARY

Treatment of acute hypertension was associated with end-organ injury

This retrospective, propensity score–matched cohort study (N = 22,834) evaluated the electronic health records of all adult patients (age > 18 years) admitted to a medicine service with a noncardiovascular diagnosis over a 1-year period at 10 Cleveland Clinic hospitals, with 1 year of follow-up data.

Exclusion criteria included hospitalization for a cardiovascular diagnosis; admission for a cerebrovascular event or acute coronary syndrome within the previous 30 days; pregnancy; length of stay of less than 2 days or more than 14 days; and lack of outpatient medication data. Patients were propensity-score matched using BP, demographic features, comorbidities, hospital shift, and time since admission. Exposure was defined as administration of IV antihypertensive medication or a new class of oral antihypertensive medication.

Continue to: Outcomes were defined...

 

 

Outcomes were defined as a temporal association between acute hypertension treatment and subsequent end-organ damage, such as AKI (serum creatinine increase ≥ 0.3 mg/dL or 1.5 × initial value [Acute Kidney Injury Network definition]), myocardial injury (elevated troponin: > 0.029 ng/mL for troponin T; > 0.045 ng/mL for troponin I), and/or stroke (indicated by discharge diagnosis, with confirmation by chart review). Monitored outcomes included stroke and myocardial infarction (MI) within 30 days of discharge and BP control up to 1 year later.

The 22,834 patients had a mean (SD) age of 65.6 (17.9) years; 12,993 (56.9%) were women, and 15,963 (69.9%) were White. Of the 17,821 (78%) who had at least 1 inpatient hypertensive systolic BP (SBP) episode, defined as an SBP ≥ 140 mm Hg, 5904 (33.1%) received a new treatment. Of those receiving a new treatment, 4378 (74.2%) received only oral treatment, and 1516 (25.7%) received at least 1 dose of IV medication with or without oral dosing.

Acute treatment of elevated BP in noncardiac inpatients was not beneficial, and treatment intensification at discharge did not improve BP control over the following year.

Using the propensity-matched sample (4520 treated for elevated BP matched to 4520 who were not treated), treated patients had higher rates of AKI (10.3% vs 7.9%; P < .001) and myocardial injury (1.2% vs 0.6%; P = .003). When assessed by SBP, nontreatment of BP was still superior up to an SBP of 199 mm Hg. At an SBP of ≥ 200 mm Hg, there was no difference in rates of AKI or MI between the treatment and nontreatment groups. There was no difference in stroke in either cohort, although the overall numbers were quite low.

Patients with and without antihypertensive intensification at discharge had similar rates of MI (0.1% vs 0.2%; P > .99) and stroke (0.5% vs 0.4%; P > .99) in a matched cohort at 30 days post discharge. At 1 year, BP control in the intensification vs no-intensification groups was nearly the same: maximum SBP was 157.2 mm Hg vs 157.8 mm Hg, respectively (P = .54) and maximum diastolic BP was 86.5 mm Hg vs 86.1 mm Hg, respectively (P = .49).

WHAT’S NEW

Previous research is confirmed in a more diverse population

Whereas previous research showed no benefit to intensification of treatment among hospitalized older male patients, this large, retrospective, propensity score–matched cohort study demonstrated the short- and long-term effects of treating acute, asymptomatic BP elevations in a younger, more generalizable population that included women. Regardless of treatment modality, there appeared to be more harm than good from treating these BP elevations.

In addition, the study appears to corroborate previous research showing that intensification of BP treatment at discharge did not lead to better outcomes.9 At the very least, the study makes a reasonable argument that treating acute BP elevations in noncardiac patients in the hospital setting is not beneficial.

CAVEATS

Impact of existing therapy could be underestimated

This study had several important limitations. First, 23% of treated participants were excluded from the propensity analysis without justification from the authors. Additionally, there was no reporting of missing data and how it was managed. The authors’ definition of treatment excluded dose intensification of existing antihypertensive therapy, which would undercount the number of treated patients. However, this could underestimate the actual harms of the acute antihypertensive therapy. The authors also included patients with atrial fibrillation and heart failure in the study population, even though they already may have been taking antihypertensive agents.

CHALLENGES TO IMPLEMENTATION

Potential delays in translating findings to patient care

Although several recent studies have shown the potential benefit of not treating asymptomatic acute BP elevations in inpatients, incorporating that information into electronic health record order sets or clinical decision support, and disseminating it to clinical end users, will take time. In the interim, despite these findings, patients may continue to receive IV or oral medications to treat acute, asymptomatic BP elevations while hospitalized for noncardiac diagnoses.

Files
References

1. Rastogi R, Sheehan MM, Hu B, et al. Treatment and outcomes of inpatient hypertension among adults with noncardiac admissions. JAMA Intern Med. 2021;181:345-352. doi: 10.1001/jamainternmed.2020.7501

2. Jacobs ZG, Najafi N, Fang MC, et al. Reducing unnecessary treatment of asymptomatic elevated blood pressure with intravenous medications on the general internal medicine wards: a quality improvement initiative. J Hosp Med. 2019;14:144-150. doi: 10.12788/jhm.3087

3. Pasik SD, Chiu S, Yang J, et al. Assess before Rx: reducing the overtreatment of asymptomatic blood pressure elevation in the inpatient setting. J Hosp Med. 2019;14:151-156. doi: 10.12788/jhm.3190

4. Campbell P, Baker WL, Bendel SD, et al. Intravenous hydralazine for blood pressure management in the hospitalized patient: its use is often unjustified. J Am Soc Hypertens. 2011;5:473-477. doi: 10.1016/j.jash.2011.07.002

5. Gauer R. Severe asymptomatic hypertension: evaluation and treatment. Am Fam Physician. 2017;95:492-500.

6. Lipari M, Moser LR, Petrovitch EA, et al. As-needed intravenous antihypertensive therapy and blood pressure control. J Hosp Med. 2016;11:193-198. doi: 10.1002/jhm.2510

7. Gaynor MF, Wright GC, Vondracek S. Retrospective review of the use of as-needed hydralazine and labetalol for the treatment of acute hypertension in hospitalized medicine patients. Ther Adv Cardiovasc Dis. 2018;12:7-15. doi: 10.1177/1753944717746613

8. Weder AB, Erickson S. Treatment of hypertension in the inpatient setting: use of intravenous labetalol and hydralazine. J Clin Hypertens (Greenwich). 2010;12:29-33. doi: 10.1111/j.1751-7176.2009.00196.x

9. Anderson TS, Jing B, Auerbach A, et al. Clinical outcomes after intensifying antihypertensive medication regimens among older adults at hospital discharge. JAMA Intern Med. 2019;179:1528-1536. doi: 10.1001/jamainternmed.2019.3007

Article PDF
Author and Disclosure Information

Madigan Army Medical Center, Joint Base LewisMcChord, WA

DEPUTY EDITOR
Anne Mounsey, MD

Department of Family Medicine, University of North Carolina, Chapel Hill

Issue
The Journal of Family Practice - 71(7)
Publications
Topics
Page Number
314-316
Sections
Files
Files
Author and Disclosure Information

Madigan Army Medical Center, Joint Base LewisMcChord, WA

DEPUTY EDITOR
Anne Mounsey, MD

Department of Family Medicine, University of North Carolina, Chapel Hill

Author and Disclosure Information

Madigan Army Medical Center, Joint Base LewisMcChord, WA

DEPUTY EDITOR
Anne Mounsey, MD

Department of Family Medicine, University of North Carolina, Chapel Hill

Article PDF
Article PDF

ILLUSTRATIVE CASE

A 48-year-old man is admitted to your family medicine service for cellulitis after failed outpatient therapy. He has presumed community-acquired methicillin-resistant Staphylococcus aureus infection of the left lower extremity and is receiving intravenous (IV) vancomycin. His BP this morning is 176/98 mm Hg, and the reading from the previous shift was 168/94 mm Hg. He is asymptomatic from this elevated BP. Based on protocol, his nurse is asking about treatment in response to the multiple elevated readings. How should you address the patient’s elevated BP, knowing that you will see him for a transition management appointment in 2 weeks?

Elevated BP is common in the adult inpatient setting. Prevalence estimates range from 25% to > 50%. Many factors can contribute to elevated BP in the acute illness setting, such as pain, anxiety, medication withdrawal, and volume status.2,3

Treatment of elevated BP in outpatients is well researched, with evidence-based guidelines for physicians. That is not the case for treatment of asymptomatic elevated BP in the inpatient setting. Most published guidance on inpatient management of acutely elevated BP recommends IV medications, such as hydralazine or labetalol, although there is limited evidence to support such recommendations. There is minimal evidence for outcomes-based benefit in treating acute elevations of inpatient BP, such as reduced myocardial injury or stroke; however, there is some evidence of adverse outcomes, such as hypotension and prolonged hospital stays.4-8

Although the possibility of intensifying antihypertensive therapy for those with known hypertension or those with presumed “new-onset” hypertension could theoretically lead to improved outcomes over the long term, there is little evidence to support this presumption. Rather, there is evidence that intensification of antihypertensive therapy at discharge is linked to short-term harms. This was demonstrated in a propensity-matched veteran cohort that included 4056 hospitalized older adults with hypertension (mean age, 77 years; 3961 men), equally split between those who received antihypertensive intensification at hospital discharge and those who did not. Within 30 days, patients receiving intensification had a higher risk of readmission (number needed to harm [NNH] = 27) and serious adverse events (NNH = 63).9

The current study aimed to put all these pieces together by quantifying the prevalence of hypertension in hospitalized patients, characterizing clinician response to patients’ acutely elevated BP, and comparing both short- and long-term outcomes in patients treated for acute BP elevations while hospitalized vs those who were not. The study also assessed the potential effects of antihypertensive intensification at discharge.

STUDY SUMMARY

Treatment of acute hypertension was associated with end-organ injury

This retrospective, propensity score–matched cohort study (N = 22,834) evaluated the electronic health records of all adult patients (age > 18 years) admitted to a medicine service with a noncardiovascular diagnosis over a 1-year period at 10 Cleveland Clinic hospitals, with 1 year of follow-up data.

Exclusion criteria included hospitalization for a cardiovascular diagnosis; admission for a cerebrovascular event or acute coronary syndrome within the previous 30 days; pregnancy; length of stay of less than 2 days or more than 14 days; and lack of outpatient medication data. Patients were propensity-score matched using BP, demographic features, comorbidities, hospital shift, and time since admission. Exposure was defined as administration of IV antihypertensive medication or a new class of oral antihypertensive medication.

Continue to: Outcomes were defined...

 

 

Outcomes were defined as a temporal association between acute hypertension treatment and subsequent end-organ damage, such as AKI (serum creatinine increase ≥ 0.3 mg/dL or 1.5 × initial value [Acute Kidney Injury Network definition]), myocardial injury (elevated troponin: > 0.029 ng/mL for troponin T; > 0.045 ng/mL for troponin I), and/or stroke (indicated by discharge diagnosis, with confirmation by chart review). Monitored outcomes included stroke and myocardial infarction (MI) within 30 days of discharge and BP control up to 1 year later.

The 22,834 patients had a mean (SD) age of 65.6 (17.9) years; 12,993 (56.9%) were women, and 15,963 (69.9%) were White. Of the 17,821 (78%) who had at least 1 inpatient hypertensive systolic BP (SBP) episode, defined as an SBP ≥ 140 mm Hg, 5904 (33.1%) received a new treatment. Of those receiving a new treatment, 4378 (74.2%) received only oral treatment, and 1516 (25.7%) received at least 1 dose of IV medication with or without oral dosing.

Acute treatment of elevated BP in noncardiac inpatients was not beneficial, and treatment intensification at discharge did not improve BP control over the following year.

Using the propensity-matched sample (4520 treated for elevated BP matched to 4520 who were not treated), treated patients had higher rates of AKI (10.3% vs 7.9%; P < .001) and myocardial injury (1.2% vs 0.6%; P = .003). When assessed by SBP, nontreatment of BP was still superior up to an SBP of 199 mm Hg. At an SBP of ≥ 200 mm Hg, there was no difference in rates of AKI or MI between the treatment and nontreatment groups. There was no difference in stroke in either cohort, although the overall numbers were quite low.

Patients with and without antihypertensive intensification at discharge had similar rates of MI (0.1% vs 0.2%; P > .99) and stroke (0.5% vs 0.4%; P > .99) in a matched cohort at 30 days post discharge. At 1 year, BP control in the intensification vs no-intensification groups was nearly the same: maximum SBP was 157.2 mm Hg vs 157.8 mm Hg, respectively (P = .54) and maximum diastolic BP was 86.5 mm Hg vs 86.1 mm Hg, respectively (P = .49).

WHAT’S NEW

Previous research is confirmed in a more diverse population

Whereas previous research showed no benefit to intensification of treatment among hospitalized older male patients, this large, retrospective, propensity score–matched cohort study demonstrated the short- and long-term effects of treating acute, asymptomatic BP elevations in a younger, more generalizable population that included women. Regardless of treatment modality, there appeared to be more harm than good from treating these BP elevations.

In addition, the study appears to corroborate previous research showing that intensification of BP treatment at discharge did not lead to better outcomes.9 At the very least, the study makes a reasonable argument that treating acute BP elevations in noncardiac patients in the hospital setting is not beneficial.

CAVEATS

Impact of existing therapy could be underestimated

This study had several important limitations. First, 23% of treated participants were excluded from the propensity analysis without justification from the authors. Additionally, there was no reporting of missing data and how it was managed. The authors’ definition of treatment excluded dose intensification of existing antihypertensive therapy, which would undercount the number of treated patients. However, this could underestimate the actual harms of the acute antihypertensive therapy. The authors also included patients with atrial fibrillation and heart failure in the study population, even though they already may have been taking antihypertensive agents.

CHALLENGES TO IMPLEMENTATION

Potential delays in translating findings to patient care

Although several recent studies have shown the potential benefit of not treating asymptomatic acute BP elevations in inpatients, incorporating that information into electronic health record order sets or clinical decision support, and disseminating it to clinical end users, will take time. In the interim, despite these findings, patients may continue to receive IV or oral medications to treat acute, asymptomatic BP elevations while hospitalized for noncardiac diagnoses.

ILLUSTRATIVE CASE

A 48-year-old man is admitted to your family medicine service for cellulitis after failed outpatient therapy. He has presumed community-acquired methicillin-resistant Staphylococcus aureus infection of the left lower extremity and is receiving intravenous (IV) vancomycin. His BP this morning is 176/98 mm Hg, and the reading from the previous shift was 168/94 mm Hg. He is asymptomatic from this elevated BP. Based on protocol, his nurse is asking about treatment in response to the multiple elevated readings. How should you address the patient’s elevated BP, knowing that you will see him for a transition management appointment in 2 weeks?

Elevated BP is common in the adult inpatient setting. Prevalence estimates range from 25% to > 50%. Many factors can contribute to elevated BP in the acute illness setting, such as pain, anxiety, medication withdrawal, and volume status.2,3

Treatment of elevated BP in outpatients is well researched, with evidence-based guidelines for physicians. That is not the case for treatment of asymptomatic elevated BP in the inpatient setting. Most published guidance on inpatient management of acutely elevated BP recommends IV medications, such as hydralazine or labetalol, although there is limited evidence to support such recommendations. There is minimal evidence for outcomes-based benefit in treating acute elevations of inpatient BP, such as reduced myocardial injury or stroke; however, there is some evidence of adverse outcomes, such as hypotension and prolonged hospital stays.4-8

Although the possibility of intensifying antihypertensive therapy for those with known hypertension or those with presumed “new-onset” hypertension could theoretically lead to improved outcomes over the long term, there is little evidence to support this presumption. Rather, there is evidence that intensification of antihypertensive therapy at discharge is linked to short-term harms. This was demonstrated in a propensity-matched veteran cohort that included 4056 hospitalized older adults with hypertension (mean age, 77 years; 3961 men), equally split between those who received antihypertensive intensification at hospital discharge and those who did not. Within 30 days, patients receiving intensification had a higher risk of readmission (number needed to harm [NNH] = 27) and serious adverse events (NNH = 63).9

The current study aimed to put all these pieces together by quantifying the prevalence of hypertension in hospitalized patients, characterizing clinician response to patients’ acutely elevated BP, and comparing both short- and long-term outcomes in patients treated for acute BP elevations while hospitalized vs those who were not. The study also assessed the potential effects of antihypertensive intensification at discharge.

STUDY SUMMARY

Treatment of acute hypertension was associated with end-organ injury

This retrospective, propensity score–matched cohort study (N = 22,834) evaluated the electronic health records of all adult patients (age > 18 years) admitted to a medicine service with a noncardiovascular diagnosis over a 1-year period at 10 Cleveland Clinic hospitals, with 1 year of follow-up data.

Exclusion criteria included hospitalization for a cardiovascular diagnosis; admission for a cerebrovascular event or acute coronary syndrome within the previous 30 days; pregnancy; length of stay of less than 2 days or more than 14 days; and lack of outpatient medication data. Patients were propensity-score matched using BP, demographic features, comorbidities, hospital shift, and time since admission. Exposure was defined as administration of IV antihypertensive medication or a new class of oral antihypertensive medication.

Continue to: Outcomes were defined...

 

 

Outcomes were defined as a temporal association between acute hypertension treatment and subsequent end-organ damage, such as AKI (serum creatinine increase ≥ 0.3 mg/dL or 1.5 × initial value [Acute Kidney Injury Network definition]), myocardial injury (elevated troponin: > 0.029 ng/mL for troponin T; > 0.045 ng/mL for troponin I), and/or stroke (indicated by discharge diagnosis, with confirmation by chart review). Monitored outcomes included stroke and myocardial infarction (MI) within 30 days of discharge and BP control up to 1 year later.

The 22,834 patients had a mean (SD) age of 65.6 (17.9) years; 12,993 (56.9%) were women, and 15,963 (69.9%) were White. Of the 17,821 (78%) who had at least 1 inpatient hypertensive systolic BP (SBP) episode, defined as an SBP ≥ 140 mm Hg, 5904 (33.1%) received a new treatment. Of those receiving a new treatment, 4378 (74.2%) received only oral treatment, and 1516 (25.7%) received at least 1 dose of IV medication with or without oral dosing.

Acute treatment of elevated BP in noncardiac inpatients was not beneficial, and treatment intensification at discharge did not improve BP control over the following year.

Using the propensity-matched sample (4520 treated for elevated BP matched to 4520 who were not treated), treated patients had higher rates of AKI (10.3% vs 7.9%; P < .001) and myocardial injury (1.2% vs 0.6%; P = .003). When assessed by SBP, nontreatment of BP was still superior up to an SBP of 199 mm Hg. At an SBP of ≥ 200 mm Hg, there was no difference in rates of AKI or MI between the treatment and nontreatment groups. There was no difference in stroke in either cohort, although the overall numbers were quite low.

Patients with and without antihypertensive intensification at discharge had similar rates of MI (0.1% vs 0.2%; P > .99) and stroke (0.5% vs 0.4%; P > .99) in a matched cohort at 30 days post discharge. At 1 year, BP control in the intensification vs no-intensification groups was nearly the same: maximum SBP was 157.2 mm Hg vs 157.8 mm Hg, respectively (P = .54) and maximum diastolic BP was 86.5 mm Hg vs 86.1 mm Hg, respectively (P = .49).

WHAT’S NEW

Previous research is confirmed in a more diverse population

Whereas previous research showed no benefit to intensification of treatment among hospitalized older male patients, this large, retrospective, propensity score–matched cohort study demonstrated the short- and long-term effects of treating acute, asymptomatic BP elevations in a younger, more generalizable population that included women. Regardless of treatment modality, there appeared to be more harm than good from treating these BP elevations.

In addition, the study appears to corroborate previous research showing that intensification of BP treatment at discharge did not lead to better outcomes.9 At the very least, the study makes a reasonable argument that treating acute BP elevations in noncardiac patients in the hospital setting is not beneficial.

CAVEATS

Impact of existing therapy could be underestimated

This study had several important limitations. First, 23% of treated participants were excluded from the propensity analysis without justification from the authors. Additionally, there was no reporting of missing data and how it was managed. The authors’ definition of treatment excluded dose intensification of existing antihypertensive therapy, which would undercount the number of treated patients. However, this could underestimate the actual harms of the acute antihypertensive therapy. The authors also included patients with atrial fibrillation and heart failure in the study population, even though they already may have been taking antihypertensive agents.

CHALLENGES TO IMPLEMENTATION

Potential delays in translating findings to patient care

Although several recent studies have shown the potential benefit of not treating asymptomatic acute BP elevations in inpatients, incorporating that information into electronic health record order sets or clinical decision support, and disseminating it to clinical end users, will take time. In the interim, despite these findings, patients may continue to receive IV or oral medications to treat acute, asymptomatic BP elevations while hospitalized for noncardiac diagnoses.

References

1. Rastogi R, Sheehan MM, Hu B, et al. Treatment and outcomes of inpatient hypertension among adults with noncardiac admissions. JAMA Intern Med. 2021;181:345-352. doi: 10.1001/jamainternmed.2020.7501

2. Jacobs ZG, Najafi N, Fang MC, et al. Reducing unnecessary treatment of asymptomatic elevated blood pressure with intravenous medications on the general internal medicine wards: a quality improvement initiative. J Hosp Med. 2019;14:144-150. doi: 10.12788/jhm.3087

3. Pasik SD, Chiu S, Yang J, et al. Assess before Rx: reducing the overtreatment of asymptomatic blood pressure elevation in the inpatient setting. J Hosp Med. 2019;14:151-156. doi: 10.12788/jhm.3190

4. Campbell P, Baker WL, Bendel SD, et al. Intravenous hydralazine for blood pressure management in the hospitalized patient: its use is often unjustified. J Am Soc Hypertens. 2011;5:473-477. doi: 10.1016/j.jash.2011.07.002

5. Gauer R. Severe asymptomatic hypertension: evaluation and treatment. Am Fam Physician. 2017;95:492-500.

6. Lipari M, Moser LR, Petrovitch EA, et al. As-needed intravenous antihypertensive therapy and blood pressure control. J Hosp Med. 2016;11:193-198. doi: 10.1002/jhm.2510

7. Gaynor MF, Wright GC, Vondracek S. Retrospective review of the use of as-needed hydralazine and labetalol for the treatment of acute hypertension in hospitalized medicine patients. Ther Adv Cardiovasc Dis. 2018;12:7-15. doi: 10.1177/1753944717746613

8. Weder AB, Erickson S. Treatment of hypertension in the inpatient setting: use of intravenous labetalol and hydralazine. J Clin Hypertens (Greenwich). 2010;12:29-33. doi: 10.1111/j.1751-7176.2009.00196.x

9. Anderson TS, Jing B, Auerbach A, et al. Clinical outcomes after intensifying antihypertensive medication regimens among older adults at hospital discharge. JAMA Intern Med. 2019;179:1528-1536. doi: 10.1001/jamainternmed.2019.3007

References

1. Rastogi R, Sheehan MM, Hu B, et al. Treatment and outcomes of inpatient hypertension among adults with noncardiac admissions. JAMA Intern Med. 2021;181:345-352. doi: 10.1001/jamainternmed.2020.7501

2. Jacobs ZG, Najafi N, Fang MC, et al. Reducing unnecessary treatment of asymptomatic elevated blood pressure with intravenous medications on the general internal medicine wards: a quality improvement initiative. J Hosp Med. 2019;14:144-150. doi: 10.12788/jhm.3087

3. Pasik SD, Chiu S, Yang J, et al. Assess before Rx: reducing the overtreatment of asymptomatic blood pressure elevation in the inpatient setting. J Hosp Med. 2019;14:151-156. doi: 10.12788/jhm.3190

4. Campbell P, Baker WL, Bendel SD, et al. Intravenous hydralazine for blood pressure management in the hospitalized patient: its use is often unjustified. J Am Soc Hypertens. 2011;5:473-477. doi: 10.1016/j.jash.2011.07.002

5. Gauer R. Severe asymptomatic hypertension: evaluation and treatment. Am Fam Physician. 2017;95:492-500.

6. Lipari M, Moser LR, Petrovitch EA, et al. As-needed intravenous antihypertensive therapy and blood pressure control. J Hosp Med. 2016;11:193-198. doi: 10.1002/jhm.2510

7. Gaynor MF, Wright GC, Vondracek S. Retrospective review of the use of as-needed hydralazine and labetalol for the treatment of acute hypertension in hospitalized medicine patients. Ther Adv Cardiovasc Dis. 2018;12:7-15. doi: 10.1177/1753944717746613

8. Weder AB, Erickson S. Treatment of hypertension in the inpatient setting: use of intravenous labetalol and hydralazine. J Clin Hypertens (Greenwich). 2010;12:29-33. doi: 10.1111/j.1751-7176.2009.00196.x

9. Anderson TS, Jing B, Auerbach A, et al. Clinical outcomes after intensifying antihypertensive medication regimens among older adults at hospital discharge. JAMA Intern Med. 2019;179:1528-1536. doi: 10.1001/jamainternmed.2019.3007

Issue
The Journal of Family Practice - 71(7)
Issue
The Journal of Family Practice - 71(7)
Page Number
314-316
Page Number
314-316
Publications
Publications
Topics
Article Type
Display Headline
Noncardiac inpatient has acute hypertension: Treat or not?
Display Headline
Noncardiac inpatient has acute hypertension: Treat or not?
Sections
PURLs Copyright
Copyright © 2022. The Family Physicians Inquiries Network. All rights reserved.
Inside the Article

PRACTICE CHANGER

Manage blood pressure (BP) elevations conservatively in patients admitted for noncardiac diagnoses, as acute hypertension treatment may increase the risk for acute kidney injury (AKI) and myocardial injury.

STRENGTH OF RECOMMENDATION

C: Based on a single, large, retrospective cohort study.1

Rastogi R, Sheehan MM, Hu B, et al. Treatment and outcomes of inpatient hypertension among adults with noncardiac admissions. JAMA Intern Med. 2021;181:345-352.

Disallow All Ads
Content Gating
No Gating (article Unlocked/Free)
Alternative CME
Disqus Comments
Default
Use ProPublica
Hide sidebar & use full width
render the right sidebar.
Conference Recap Checkbox
Not Conference Recap
Clinical Edge
Display the Slideshow in this Article
Medscape Article
Display survey writer
Reuters content
Disable Inline Native ads
WebMD Article
Article PDF Media
Media Files

Can early introduction of gluten reduce risk of celiac disease?

Article Type
Changed
Tue, 07/26/2022 - 10:47
Display Headline
Can early introduction of gluten reduce risk of celiac disease?

ILLUSTRATIVE CASE

You are seeing a 2-month-old female infant for a routine well-child visit. The birth history was unremarkable. The infant is meeting appropriate developmental milestones. Growth is appropriate at the 40th percentile. The infant is exclusively breastfed. The parents report that they have heard confusing information about when to introduce solid foods, including bread, to their child’s diet. There is no known family history of CD. What anticipatory guidance can you offer regarding gluten introduction and the risk of CD?

CD is an inflammatory disease of the small intestine caused by an immune-based reaction to dietary gluten. The worldwide incidence of CD in children younger than 15 years is 21.3 per 100,000 person-years; this incidence has increased by 7.5% per year over the past several decades.2 CD has a range of both gastrointestinal and nongastrointestinal manifestations, including diarrhea, weight loss, abdominal pain, abnormal liver function test results, and iron deficiency anemia.

Diagnosis of CD in adults is based on a combination of clinical symptoms, elevated levels of immunoglobulin A anti-tissue transglutaminase antibody (tTG-IgA), and biopsy-confirmed villous atrophy of the duodenum on upper endoscopy.3 European pediatric guidelines suggest that use of certain criteria, including very high results of tTG-IgA antibody testing (> 10 times the upper limit of normal), can help to avoid endoscopic biopsies and/or human leukocyte antigens (HLA) testing for diagnosis in children.4

The mainstay of CD management is strict adherence to a gluten-free diet.3 Because this can be difficult, and yield an incomplete disease response, emphasis has been placed on primary prevention by modifying introduction of dietary gluten. Multiple prior studies examining the risk of CD have failed to demonstrate a significant association between timing of gluten introduction and development of CD among high-risk infants (eg, those with HLA-DR3 alleles or first-degree relatives with CD or type 1 diabetes).5-7 A 2016 meta-analysis concluded that there was not enough evidence to support early introduction of gluten (at 4-6 months).8 RCTs have not previously been conducted to examine the timing of gluten introduction on CD prevalence for infants at average risk, using age-appropriate doses of gluten prior to age 6 months.

Current dietary guidelines in the United States and the United Kingdom recommend introduction of nutrient-dense foods, including potentially allergenic foods, at about age 6 months to complement human milk or infant formula feedings.9,10 These guidelines do not specify the exact timing or quantity of gluten- containing food introduction for infants. A 2016 position paper by the European Society for Pediatric Gastroenterology, Hepatology, and Nutrition indicated that gluten could be introduced into the infant’s diet any time between 4 and 12 months. They did indicate that the amount of gluten introduced into the diet should be low to start and then increased, and that infants at high risk for CD should wait longer for gluten introduction (4 vs 6 months or 6 vs 12 months).11

STUDY SUMMARY

Gluten introduced at 4 months may be linked to lower occurrence of CD

The Enquiring About Tolerance (EAT) Study was an open-label RCT (N = 1303) with children from the general population in England and Wales. The EAT Study sought to test the prevention of food allergy by introducing allergenic foods to infants at age 4 months compared with exclusively breastfeeding until age 6 months. The median age at enrollment was 3.4 months, but allergenic food was not started until age 4 months.1,12 Most patients were White (84.%-85.4%) and lived in an urban area (77.3%-77.4%). The mean gestational age at delivery was 39.7 to 39.9 weeks.12

Infants were exclusively breastfed until age 13 weeks, at which time they were randomized into an early introduction group (EIG) or a standard introduction group (SIG). In addition to breast milk, infants in the EIG consumed 6 allergenic foods (peanut, sesame, hen’s egg, cow’s milk, cod fish, and wheat [gluten]) in a specified pattern per protocol, starting at age 4 months. Wheat (gluten) was introduced during Week 5 of the EIG protocol (median age, 20.6 weeks).12 The recommended minimum dose of gluten was 3.2 g/wk from age 16 weeks, or 4 g/wk of wheat protein (given as 2 cereal biscuits or the equivalent). Infants in the SIG avoided allergenic foods, following UK infant feeding recommendations for exclusive breastfeeding until about age 6 months. The EIG had a significantly higher rate of cesarean births than the SIG, but the study groups were otherwise balanced.13

Continue to: Families completed monthly...

 

 

Families completed monthly questionnaires on infant gluten intake and symptoms (eg, gastrointestinal, fatigue) through age 1 year, and then every 3 months through age 3 years. All children were tested for anti-transglutaminase type 2 (anti-TG2) antibodies at age 3 years as a screen for CD. Children with antibody levels > 20 IU/L were referred to independent gastroenterologists for further evaluation, which could include HLA (DQ-2/DQ-8) testing and biopsy in accordance with current European diagnostic guidelines.4

Introducing gluten as a complement to breast milk or infant formula from age 4 months may reduce the risk of celiac disease at age 3 years.

In an intention-to-treat analysis for the primary outcome, 595 children in the SIG (91.4%) and 567 in the EIG (87.0%) were included. Between ages 4 and 6 months, the mean (SD) quantity of gluten consumed in the SIG was 0.49 (1.40) g/wk; in the EIG, the mean quantity was 2.66 (1.85) g/wk (P < .001). At age 3 years, of a total of 1004 children tested for anti-TG2 antibodies, 9 had anti-TG2 levels requiring referral (7 in the SIG and 2 in the EIG). A diagnosis of CD was confirmed in 7 of 516 children in the SIG (1.4%) vs none of the 488 children in the EIG (P = .02). Using bootstrap resampling, the risk difference between the groups was 1.4% (95% CI, 0.6%-2.6%).

WHAT’S NEW

Findings have potential to change nutritional guidance

This study demonstrated that introduction of age-appropriate portions of gluten-containing products at age 4 months, in addition to breast milk, may reduce the risk of CD at 3 years in children at average risk. This finding has the potential to change anticipatory guidance given to parents regarding infant nutrition recommendations.

CAVEATS

More studies needed to confirm prevention vs delay of CD

The homogeneous study population may limit generalizability. Infants in this study were from England and Wales (84.3% were White), born at term, and were exclusively breastfed until age 13 weeks. Further studies are required to determine whether these findings can be applied to infants who are no longer breastfeeding, are more racially diverse, or are preterm in gestational age at birth. Additionally, the study followed the participants only until age 3 years. Given that the onset of CD after this age is likely, further research is needed to support that CD is truly prevented rather than delayed.

CHALLENGES TO IMPLEMENTATION

Guidance on allergen introduction may be unclear

The EAT Study protocol required parents in the EIG to sequentially introduce a minimum amount of the 6 allergenic foods specified. Only 42% of the EIG cohort reported adherence to the protocol.12 It is unclear how important this specific regimen is to the study results and whether introduction of all 6 allergenic foods simultaneously modifies the immune response to gluten. Therefore, there may be challenges to implementation if physicians do not know how to provide anticipatory guidance on the appropriate steps for allergen introduction.

References

1. Logan K, Perkin MR, Marrs T, et al. Early gluten introduction and celiac disease in the EAT Study: a prespecified analysis of the EAT randomized clinical trial. JAMA Pediatr. 2020;174:1041-1047. doi: 10.1001/jamapediatrics.2020.2893

2. King JA, Jeong J, Underwood FE, et al. Incidence of celiac disease is increasing over time: a systematic review and meta-analysis. Am J Gastroenterol. 2020;115:507-525. doi: 10.14309/ajg.0000000000000523

3. Rubio-Tapia A, Hill ID, Kelly CP, et al; American College of Gastroenterology. ACG clinical guidelines: diagnosis and management of celiac disease. Am J Gastroenterol. 2013;108:656-676; quiz 677. doi: 10.1038/ajg.2013.79

4. Husby S, Koletzko S, Korponay-Szabó I, et al. European Society Paediatric Gastroenterology, Hepatology and Nutrition guidelines for diagnosing coeliac disease 2020. J Pediatr Gastroenterol Nutr. 2020;70:141-156. doi: 10.1097/MPG.0000000000002497

5. Vriezinga SL, Auricchio R, Bravi E, et al. Randomized feeding intervention in infants at high risk for celiac disease. N Engl J Med. 2014;371:1304-1315. doi: 10.1056/NEJMoa1404172

6. Beyerlein A, Chmiel R, Hummel S, et al. Timing of gluten introduction and islet autoimmunity in young children: updated results from the BABYDIET study. Diabetes Care. 2014;37:e194-e195. doi: 10.2337/dc14-1208

7. Lionetti E, Castellaneta S, Francavilla R, et al; SIGENP (Italian Society of Pediatric Gastroenterology, Hepatology, and Nutrition) Working Group on Weaning and CD Risk. Introduction of gluten, HLA status, and the risk of celiac disease in children. N Engl J Med. 2014;371:1295-1303. doi: 10.1056/NEJMoa1400697

8. Pinto-Sánchez MI, Verdu EF, Liu E, et al. Gluten introduction to infant feeding and risk of celiac disease: systematic review and meta-analysis. J Pediatr. 2016;168:132-143.e3. doi: 10.1016/j.jpeds.2015.09.032

9. US Department of Agriculture, US Department of Health and Human Services. Dietary Guidelines for Americans, 2020-2025. 9th ed. December 2020. Accessed June 8, 2022. www.dietaryguidelines.gov/sites/default/files/2021-03/Dietary_Guidelines_for_Americans-2020-2025.pdf

10. NHS. Food allergies in babies and young children. Last reviewed November 5, 2021. Accessed June 8, 2022. www.nhs.uk/conditions/baby/weaning-and-feeding/food-allergies-in-babies-and-young-children/

11. Szajewska H, Shamir R, Mearin L, et al. Gluten introduction and the risk of coeliac disease: a position paper by the European Society for Pediatric Gastroenterology, Hepatology, and Nutrition. J Pediatr Gastroenterol Nutr. 2016;62:507-513. doi: 10.1097/MPG.0000000000001105

12. Perkin MR, Logan K, Marrs T, et al; EAT Study Team. Enquiring About Tolerance (EAT) study: feasibility of an early allergenic food introduction regimen. J Allergy Clin Immunol. 2016;137:1477-1486.e8. doi: 10.1016/j.jaci.2015.12.1322

13. Perkin MR, Logan K, Tseng A, et al; EAT Study Team. Randomized trial of introduction of allergenic foods in breast-fed infants. N Engl J Med. 2016;374:1733-1743. doi: 10.1056/NEJMoa1514210

Article PDF
Author and Disclosure Information

Madigan Army Medical Center, Joint Base Lewis-McChord, WA

DEPUTY EDITOR
Rebecca Mullen, MD, MPH

University of Colorado Department of Family Medicine, Denver

Issue
The Journal of Family Practice - 71(6)
Publications
Topics
Page Number
E4-E6
Sections
Author and Disclosure Information

Madigan Army Medical Center, Joint Base Lewis-McChord, WA

DEPUTY EDITOR
Rebecca Mullen, MD, MPH

University of Colorado Department of Family Medicine, Denver

Author and Disclosure Information

Madigan Army Medical Center, Joint Base Lewis-McChord, WA

DEPUTY EDITOR
Rebecca Mullen, MD, MPH

University of Colorado Department of Family Medicine, Denver

Article PDF
Article PDF

ILLUSTRATIVE CASE

You are seeing a 2-month-old female infant for a routine well-child visit. The birth history was unremarkable. The infant is meeting appropriate developmental milestones. Growth is appropriate at the 40th percentile. The infant is exclusively breastfed. The parents report that they have heard confusing information about when to introduce solid foods, including bread, to their child’s diet. There is no known family history of CD. What anticipatory guidance can you offer regarding gluten introduction and the risk of CD?

CD is an inflammatory disease of the small intestine caused by an immune-based reaction to dietary gluten. The worldwide incidence of CD in children younger than 15 years is 21.3 per 100,000 person-years; this incidence has increased by 7.5% per year over the past several decades.2 CD has a range of both gastrointestinal and nongastrointestinal manifestations, including diarrhea, weight loss, abdominal pain, abnormal liver function test results, and iron deficiency anemia.

Diagnosis of CD in adults is based on a combination of clinical symptoms, elevated levels of immunoglobulin A anti-tissue transglutaminase antibody (tTG-IgA), and biopsy-confirmed villous atrophy of the duodenum on upper endoscopy.3 European pediatric guidelines suggest that use of certain criteria, including very high results of tTG-IgA antibody testing (> 10 times the upper limit of normal), can help to avoid endoscopic biopsies and/or human leukocyte antigens (HLA) testing for diagnosis in children.4

The mainstay of CD management is strict adherence to a gluten-free diet.3 Because this can be difficult, and yield an incomplete disease response, emphasis has been placed on primary prevention by modifying introduction of dietary gluten. Multiple prior studies examining the risk of CD have failed to demonstrate a significant association between timing of gluten introduction and development of CD among high-risk infants (eg, those with HLA-DR3 alleles or first-degree relatives with CD or type 1 diabetes).5-7 A 2016 meta-analysis concluded that there was not enough evidence to support early introduction of gluten (at 4-6 months).8 RCTs have not previously been conducted to examine the timing of gluten introduction on CD prevalence for infants at average risk, using age-appropriate doses of gluten prior to age 6 months.

Current dietary guidelines in the United States and the United Kingdom recommend introduction of nutrient-dense foods, including potentially allergenic foods, at about age 6 months to complement human milk or infant formula feedings.9,10 These guidelines do not specify the exact timing or quantity of gluten- containing food introduction for infants. A 2016 position paper by the European Society for Pediatric Gastroenterology, Hepatology, and Nutrition indicated that gluten could be introduced into the infant’s diet any time between 4 and 12 months. They did indicate that the amount of gluten introduced into the diet should be low to start and then increased, and that infants at high risk for CD should wait longer for gluten introduction (4 vs 6 months or 6 vs 12 months).11

STUDY SUMMARY

Gluten introduced at 4 months may be linked to lower occurrence of CD

The Enquiring About Tolerance (EAT) Study was an open-label RCT (N = 1303) with children from the general population in England and Wales. The EAT Study sought to test the prevention of food allergy by introducing allergenic foods to infants at age 4 months compared with exclusively breastfeeding until age 6 months. The median age at enrollment was 3.4 months, but allergenic food was not started until age 4 months.1,12 Most patients were White (84.%-85.4%) and lived in an urban area (77.3%-77.4%). The mean gestational age at delivery was 39.7 to 39.9 weeks.12

Infants were exclusively breastfed until age 13 weeks, at which time they were randomized into an early introduction group (EIG) or a standard introduction group (SIG). In addition to breast milk, infants in the EIG consumed 6 allergenic foods (peanut, sesame, hen’s egg, cow’s milk, cod fish, and wheat [gluten]) in a specified pattern per protocol, starting at age 4 months. Wheat (gluten) was introduced during Week 5 of the EIG protocol (median age, 20.6 weeks).12 The recommended minimum dose of gluten was 3.2 g/wk from age 16 weeks, or 4 g/wk of wheat protein (given as 2 cereal biscuits or the equivalent). Infants in the SIG avoided allergenic foods, following UK infant feeding recommendations for exclusive breastfeeding until about age 6 months. The EIG had a significantly higher rate of cesarean births than the SIG, but the study groups were otherwise balanced.13

Continue to: Families completed monthly...

 

 

Families completed monthly questionnaires on infant gluten intake and symptoms (eg, gastrointestinal, fatigue) through age 1 year, and then every 3 months through age 3 years. All children were tested for anti-transglutaminase type 2 (anti-TG2) antibodies at age 3 years as a screen for CD. Children with antibody levels > 20 IU/L were referred to independent gastroenterologists for further evaluation, which could include HLA (DQ-2/DQ-8) testing and biopsy in accordance with current European diagnostic guidelines.4

Introducing gluten as a complement to breast milk or infant formula from age 4 months may reduce the risk of celiac disease at age 3 years.

In an intention-to-treat analysis for the primary outcome, 595 children in the SIG (91.4%) and 567 in the EIG (87.0%) were included. Between ages 4 and 6 months, the mean (SD) quantity of gluten consumed in the SIG was 0.49 (1.40) g/wk; in the EIG, the mean quantity was 2.66 (1.85) g/wk (P < .001). At age 3 years, of a total of 1004 children tested for anti-TG2 antibodies, 9 had anti-TG2 levels requiring referral (7 in the SIG and 2 in the EIG). A diagnosis of CD was confirmed in 7 of 516 children in the SIG (1.4%) vs none of the 488 children in the EIG (P = .02). Using bootstrap resampling, the risk difference between the groups was 1.4% (95% CI, 0.6%-2.6%).

WHAT’S NEW

Findings have potential to change nutritional guidance

This study demonstrated that introduction of age-appropriate portions of gluten-containing products at age 4 months, in addition to breast milk, may reduce the risk of CD at 3 years in children at average risk. This finding has the potential to change anticipatory guidance given to parents regarding infant nutrition recommendations.

CAVEATS

More studies needed to confirm prevention vs delay of CD

The homogeneous study population may limit generalizability. Infants in this study were from England and Wales (84.3% were White), born at term, and were exclusively breastfed until age 13 weeks. Further studies are required to determine whether these findings can be applied to infants who are no longer breastfeeding, are more racially diverse, or are preterm in gestational age at birth. Additionally, the study followed the participants only until age 3 years. Given that the onset of CD after this age is likely, further research is needed to support that CD is truly prevented rather than delayed.

CHALLENGES TO IMPLEMENTATION

Guidance on allergen introduction may be unclear

The EAT Study protocol required parents in the EIG to sequentially introduce a minimum amount of the 6 allergenic foods specified. Only 42% of the EIG cohort reported adherence to the protocol.12 It is unclear how important this specific regimen is to the study results and whether introduction of all 6 allergenic foods simultaneously modifies the immune response to gluten. Therefore, there may be challenges to implementation if physicians do not know how to provide anticipatory guidance on the appropriate steps for allergen introduction.

ILLUSTRATIVE CASE

You are seeing a 2-month-old female infant for a routine well-child visit. The birth history was unremarkable. The infant is meeting appropriate developmental milestones. Growth is appropriate at the 40th percentile. The infant is exclusively breastfed. The parents report that they have heard confusing information about when to introduce solid foods, including bread, to their child’s diet. There is no known family history of CD. What anticipatory guidance can you offer regarding gluten introduction and the risk of CD?

CD is an inflammatory disease of the small intestine caused by an immune-based reaction to dietary gluten. The worldwide incidence of CD in children younger than 15 years is 21.3 per 100,000 person-years; this incidence has increased by 7.5% per year over the past several decades.2 CD has a range of both gastrointestinal and nongastrointestinal manifestations, including diarrhea, weight loss, abdominal pain, abnormal liver function test results, and iron deficiency anemia.

Diagnosis of CD in adults is based on a combination of clinical symptoms, elevated levels of immunoglobulin A anti-tissue transglutaminase antibody (tTG-IgA), and biopsy-confirmed villous atrophy of the duodenum on upper endoscopy.3 European pediatric guidelines suggest that use of certain criteria, including very high results of tTG-IgA antibody testing (> 10 times the upper limit of normal), can help to avoid endoscopic biopsies and/or human leukocyte antigens (HLA) testing for diagnosis in children.4

The mainstay of CD management is strict adherence to a gluten-free diet.3 Because this can be difficult, and yield an incomplete disease response, emphasis has been placed on primary prevention by modifying introduction of dietary gluten. Multiple prior studies examining the risk of CD have failed to demonstrate a significant association between timing of gluten introduction and development of CD among high-risk infants (eg, those with HLA-DR3 alleles or first-degree relatives with CD or type 1 diabetes).5-7 A 2016 meta-analysis concluded that there was not enough evidence to support early introduction of gluten (at 4-6 months).8 RCTs have not previously been conducted to examine the timing of gluten introduction on CD prevalence for infants at average risk, using age-appropriate doses of gluten prior to age 6 months.

Current dietary guidelines in the United States and the United Kingdom recommend introduction of nutrient-dense foods, including potentially allergenic foods, at about age 6 months to complement human milk or infant formula feedings.9,10 These guidelines do not specify the exact timing or quantity of gluten- containing food introduction for infants. A 2016 position paper by the European Society for Pediatric Gastroenterology, Hepatology, and Nutrition indicated that gluten could be introduced into the infant’s diet any time between 4 and 12 months. They did indicate that the amount of gluten introduced into the diet should be low to start and then increased, and that infants at high risk for CD should wait longer for gluten introduction (4 vs 6 months or 6 vs 12 months).11

STUDY SUMMARY

Gluten introduced at 4 months may be linked to lower occurrence of CD

The Enquiring About Tolerance (EAT) Study was an open-label RCT (N = 1303) with children from the general population in England and Wales. The EAT Study sought to test the prevention of food allergy by introducing allergenic foods to infants at age 4 months compared with exclusively breastfeeding until age 6 months. The median age at enrollment was 3.4 months, but allergenic food was not started until age 4 months.1,12 Most patients were White (84.%-85.4%) and lived in an urban area (77.3%-77.4%). The mean gestational age at delivery was 39.7 to 39.9 weeks.12

Infants were exclusively breastfed until age 13 weeks, at which time they were randomized into an early introduction group (EIG) or a standard introduction group (SIG). In addition to breast milk, infants in the EIG consumed 6 allergenic foods (peanut, sesame, hen’s egg, cow’s milk, cod fish, and wheat [gluten]) in a specified pattern per protocol, starting at age 4 months. Wheat (gluten) was introduced during Week 5 of the EIG protocol (median age, 20.6 weeks).12 The recommended minimum dose of gluten was 3.2 g/wk from age 16 weeks, or 4 g/wk of wheat protein (given as 2 cereal biscuits or the equivalent). Infants in the SIG avoided allergenic foods, following UK infant feeding recommendations for exclusive breastfeeding until about age 6 months. The EIG had a significantly higher rate of cesarean births than the SIG, but the study groups were otherwise balanced.13

Continue to: Families completed monthly...

 

 

Families completed monthly questionnaires on infant gluten intake and symptoms (eg, gastrointestinal, fatigue) through age 1 year, and then every 3 months through age 3 years. All children were tested for anti-transglutaminase type 2 (anti-TG2) antibodies at age 3 years as a screen for CD. Children with antibody levels > 20 IU/L were referred to independent gastroenterologists for further evaluation, which could include HLA (DQ-2/DQ-8) testing and biopsy in accordance with current European diagnostic guidelines.4

Introducing gluten as a complement to breast milk or infant formula from age 4 months may reduce the risk of celiac disease at age 3 years.

In an intention-to-treat analysis for the primary outcome, 595 children in the SIG (91.4%) and 567 in the EIG (87.0%) were included. Between ages 4 and 6 months, the mean (SD) quantity of gluten consumed in the SIG was 0.49 (1.40) g/wk; in the EIG, the mean quantity was 2.66 (1.85) g/wk (P < .001). At age 3 years, of a total of 1004 children tested for anti-TG2 antibodies, 9 had anti-TG2 levels requiring referral (7 in the SIG and 2 in the EIG). A diagnosis of CD was confirmed in 7 of 516 children in the SIG (1.4%) vs none of the 488 children in the EIG (P = .02). Using bootstrap resampling, the risk difference between the groups was 1.4% (95% CI, 0.6%-2.6%).

WHAT’S NEW

Findings have potential to change nutritional guidance

This study demonstrated that introduction of age-appropriate portions of gluten-containing products at age 4 months, in addition to breast milk, may reduce the risk of CD at 3 years in children at average risk. This finding has the potential to change anticipatory guidance given to parents regarding infant nutrition recommendations.

CAVEATS

More studies needed to confirm prevention vs delay of CD

The homogeneous study population may limit generalizability. Infants in this study were from England and Wales (84.3% were White), born at term, and were exclusively breastfed until age 13 weeks. Further studies are required to determine whether these findings can be applied to infants who are no longer breastfeeding, are more racially diverse, or are preterm in gestational age at birth. Additionally, the study followed the participants only until age 3 years. Given that the onset of CD after this age is likely, further research is needed to support that CD is truly prevented rather than delayed.

CHALLENGES TO IMPLEMENTATION

Guidance on allergen introduction may be unclear

The EAT Study protocol required parents in the EIG to sequentially introduce a minimum amount of the 6 allergenic foods specified. Only 42% of the EIG cohort reported adherence to the protocol.12 It is unclear how important this specific regimen is to the study results and whether introduction of all 6 allergenic foods simultaneously modifies the immune response to gluten. Therefore, there may be challenges to implementation if physicians do not know how to provide anticipatory guidance on the appropriate steps for allergen introduction.

References

1. Logan K, Perkin MR, Marrs T, et al. Early gluten introduction and celiac disease in the EAT Study: a prespecified analysis of the EAT randomized clinical trial. JAMA Pediatr. 2020;174:1041-1047. doi: 10.1001/jamapediatrics.2020.2893

2. King JA, Jeong J, Underwood FE, et al. Incidence of celiac disease is increasing over time: a systematic review and meta-analysis. Am J Gastroenterol. 2020;115:507-525. doi: 10.14309/ajg.0000000000000523

3. Rubio-Tapia A, Hill ID, Kelly CP, et al; American College of Gastroenterology. ACG clinical guidelines: diagnosis and management of celiac disease. Am J Gastroenterol. 2013;108:656-676; quiz 677. doi: 10.1038/ajg.2013.79

4. Husby S, Koletzko S, Korponay-Szabó I, et al. European Society Paediatric Gastroenterology, Hepatology and Nutrition guidelines for diagnosing coeliac disease 2020. J Pediatr Gastroenterol Nutr. 2020;70:141-156. doi: 10.1097/MPG.0000000000002497

5. Vriezinga SL, Auricchio R, Bravi E, et al. Randomized feeding intervention in infants at high risk for celiac disease. N Engl J Med. 2014;371:1304-1315. doi: 10.1056/NEJMoa1404172

6. Beyerlein A, Chmiel R, Hummel S, et al. Timing of gluten introduction and islet autoimmunity in young children: updated results from the BABYDIET study. Diabetes Care. 2014;37:e194-e195. doi: 10.2337/dc14-1208

7. Lionetti E, Castellaneta S, Francavilla R, et al; SIGENP (Italian Society of Pediatric Gastroenterology, Hepatology, and Nutrition) Working Group on Weaning and CD Risk. Introduction of gluten, HLA status, and the risk of celiac disease in children. N Engl J Med. 2014;371:1295-1303. doi: 10.1056/NEJMoa1400697

8. Pinto-Sánchez MI, Verdu EF, Liu E, et al. Gluten introduction to infant feeding and risk of celiac disease: systematic review and meta-analysis. J Pediatr. 2016;168:132-143.e3. doi: 10.1016/j.jpeds.2015.09.032

9. US Department of Agriculture, US Department of Health and Human Services. Dietary Guidelines for Americans, 2020-2025. 9th ed. December 2020. Accessed June 8, 2022. www.dietaryguidelines.gov/sites/default/files/2021-03/Dietary_Guidelines_for_Americans-2020-2025.pdf

10. NHS. Food allergies in babies and young children. Last reviewed November 5, 2021. Accessed June 8, 2022. www.nhs.uk/conditions/baby/weaning-and-feeding/food-allergies-in-babies-and-young-children/

11. Szajewska H, Shamir R, Mearin L, et al. Gluten introduction and the risk of coeliac disease: a position paper by the European Society for Pediatric Gastroenterology, Hepatology, and Nutrition. J Pediatr Gastroenterol Nutr. 2016;62:507-513. doi: 10.1097/MPG.0000000000001105

12. Perkin MR, Logan K, Marrs T, et al; EAT Study Team. Enquiring About Tolerance (EAT) study: feasibility of an early allergenic food introduction regimen. J Allergy Clin Immunol. 2016;137:1477-1486.e8. doi: 10.1016/j.jaci.2015.12.1322

13. Perkin MR, Logan K, Tseng A, et al; EAT Study Team. Randomized trial of introduction of allergenic foods in breast-fed infants. N Engl J Med. 2016;374:1733-1743. doi: 10.1056/NEJMoa1514210

References

1. Logan K, Perkin MR, Marrs T, et al. Early gluten introduction and celiac disease in the EAT Study: a prespecified analysis of the EAT randomized clinical trial. JAMA Pediatr. 2020;174:1041-1047. doi: 10.1001/jamapediatrics.2020.2893

2. King JA, Jeong J, Underwood FE, et al. Incidence of celiac disease is increasing over time: a systematic review and meta-analysis. Am J Gastroenterol. 2020;115:507-525. doi: 10.14309/ajg.0000000000000523

3. Rubio-Tapia A, Hill ID, Kelly CP, et al; American College of Gastroenterology. ACG clinical guidelines: diagnosis and management of celiac disease. Am J Gastroenterol. 2013;108:656-676; quiz 677. doi: 10.1038/ajg.2013.79

4. Husby S, Koletzko S, Korponay-Szabó I, et al. European Society Paediatric Gastroenterology, Hepatology and Nutrition guidelines for diagnosing coeliac disease 2020. J Pediatr Gastroenterol Nutr. 2020;70:141-156. doi: 10.1097/MPG.0000000000002497

5. Vriezinga SL, Auricchio R, Bravi E, et al. Randomized feeding intervention in infants at high risk for celiac disease. N Engl J Med. 2014;371:1304-1315. doi: 10.1056/NEJMoa1404172

6. Beyerlein A, Chmiel R, Hummel S, et al. Timing of gluten introduction and islet autoimmunity in young children: updated results from the BABYDIET study. Diabetes Care. 2014;37:e194-e195. doi: 10.2337/dc14-1208

7. Lionetti E, Castellaneta S, Francavilla R, et al; SIGENP (Italian Society of Pediatric Gastroenterology, Hepatology, and Nutrition) Working Group on Weaning and CD Risk. Introduction of gluten, HLA status, and the risk of celiac disease in children. N Engl J Med. 2014;371:1295-1303. doi: 10.1056/NEJMoa1400697

8. Pinto-Sánchez MI, Verdu EF, Liu E, et al. Gluten introduction to infant feeding and risk of celiac disease: systematic review and meta-analysis. J Pediatr. 2016;168:132-143.e3. doi: 10.1016/j.jpeds.2015.09.032

9. US Department of Agriculture, US Department of Health and Human Services. Dietary Guidelines for Americans, 2020-2025. 9th ed. December 2020. Accessed June 8, 2022. www.dietaryguidelines.gov/sites/default/files/2021-03/Dietary_Guidelines_for_Americans-2020-2025.pdf

10. NHS. Food allergies in babies and young children. Last reviewed November 5, 2021. Accessed June 8, 2022. www.nhs.uk/conditions/baby/weaning-and-feeding/food-allergies-in-babies-and-young-children/

11. Szajewska H, Shamir R, Mearin L, et al. Gluten introduction and the risk of coeliac disease: a position paper by the European Society for Pediatric Gastroenterology, Hepatology, and Nutrition. J Pediatr Gastroenterol Nutr. 2016;62:507-513. doi: 10.1097/MPG.0000000000001105

12. Perkin MR, Logan K, Marrs T, et al; EAT Study Team. Enquiring About Tolerance (EAT) study: feasibility of an early allergenic food introduction regimen. J Allergy Clin Immunol. 2016;137:1477-1486.e8. doi: 10.1016/j.jaci.2015.12.1322

13. Perkin MR, Logan K, Tseng A, et al; EAT Study Team. Randomized trial of introduction of allergenic foods in breast-fed infants. N Engl J Med. 2016;374:1733-1743. doi: 10.1056/NEJMoa1514210

Issue
The Journal of Family Practice - 71(6)
Issue
The Journal of Family Practice - 71(6)
Page Number
E4-E6
Page Number
E4-E6
Publications
Publications
Topics
Article Type
Display Headline
Can early introduction of gluten reduce risk of celiac disease?
Display Headline
Can early introduction of gluten reduce risk of celiac disease?
Sections
PURLs Copyright
Copyright © 2022. The Family Physicians Inquiries Network. All rights reserved.
Inside the Article

PRACTICE CHANGER

Consider introducing gluten (wheat) in addition to breast milk or infant formula from age 4 months to potentially reduce the risk of celiac disease (CD) at age 3 years.1

STRENGTH OF RECOMMENDATION

B: Based on a single randomized controlled trial (RCT) with a patient-oriented outcome of CD diagnosis.1

Logan K, Perkin MR, Marrs T, et al. Early gluten introduction and celiac disease in the EAT Study: a prespecified analysis of the EAT randomized clinical trial. JAMA Pediatr. 2020;174:1041-1047.

Disallow All Ads
Content Gating
No Gating (article Unlocked/Free)
Alternative CME
Disqus Comments
Default
Use ProPublica
Hide sidebar & use full width
render the right sidebar.
Conference Recap Checkbox
Not Conference Recap
Clinical Edge
Display the Slideshow in this Article
Medscape Article
Display survey writer
Reuters content
Disable Inline Native ads
WebMD Article
Article PDF Media

Colchicine may decrease cardiovascular events in patients with coronary artery disease

Article Type
Changed
Tue, 07/26/2022 - 11:08
Display Headline
Colchicine may decrease cardiovascular events in patients with coronary artery disease

ILLUSTRATIVE CASE

A 62-year-old woman with a past medical history of type 2 diabetes, hyperlipidemia, hypertension, and remote myocardial infarction (MI) presents to her primary care office for a preventive visit. She is a nonsmoker and has been taking her daily medications as prescribed, including an angiotensin-converting enzyme inhibitor, high-intensity statin, and aspirin. Her diabetes is well controlled. What else would you consider recommending to decrease this patient’s risk for future CVEs?

Cardiovascular disease (CVD) is a major contributor to morbidity and mortality, affecting more than 50% of patients older than 60.2 Despite control of risk factors with standard treatment modalities, patients with established CVD remain at high risk for future events, which makes elucidating and targeting other causative pathways essential.3

Inflammation has been identified as a key player in the development and progression of atherosclerosis and its downstream effects, with increased inflammatory markers correlating with increased risk for CVEs.4 Due to these findings, anti-inflammatory treatments have been under investigation as agents to further reduce risk for CVEs. In 1 such trial, the Canakinumab Antiinflammatory Thrombosis Outcome Study (CANTOS), patients with MI and elevated C-reactive protein levels treated with the interleukin-1 beta inhibitor canakinumab showed reduced risk for future CVEs compared to those receiving placebo.5 However, due to canakinumab’s high cost, inconvenient subcutaneous administration, and increased incidence of fatal infections, other agents are under investigation.

Colchicine is a potent anti-inflammatory agent, with approval in the United States for treatment of gout and familial Mediterranean fever. It works broadly to reduce inflammation by disrupting tubulin polymerization.6,7 Colchicine decreases interleukin-1 beta production through inactivation of the NLRP3 inflammasome pathway, which has been associated with the inflammatory component driving atherosclerotic plaque progression and instability.5,8 Colchicine’s oral administration, relative cost-effectiveness, and safety profile make it an attractive option for potential use in secondary prevention of CVEs.

The Low-Dose Colchicine (LoDoCo) trial, published in 2013, demonstrated a reduction in CVEs in those with CVD taking guideline-directed medical therapy (GDMT) plus colchicine 0.5 mg/d, compared with those taking GDMT alone.9 However, the LoDoCo study enrolled only 532 patients and was not placebo controlled. The Colchicine Cardiovascular Outcomes Trial (COLCOT), published in 2019, was a randomized, ­double-blind, placebo-controlled trial that aimed to further evaluate the effects of colchicine on CVEs on a larger scale and to assess its longer-term safety.10 In this study, the colchicine group had a significantly lower risk of CVEs vs placebo, with a comparable safety profile.10

STUDY SUMMARY

Fewer CVEs occurred when colchicine was added to the regimen

The randomized, multicenter, double-blind Low Dose Colchicine 2 (LoDoCo2) trial evaluated whether colchicine 0.5 mg daily reduces CV death, spontaneous (nonprocedural) MI, ischemic stroke, or ischemia-driven coronary revascularization in patients with chronic CAD (composite primary endpoint). This trial included 5522 patients, ages 35 to 82, in Australia and the Netherlands. Patients were eligible to participate if they had evidence of CAD by invasive coronary angiography, coronary calcium score, or computed tomography angiography, as well as evidence of clinical stability for 6 months. Exclusion criteria included moderate-to-severe renal impairment, severe heart failure, severe valvular disease, or intolerance to colchicine.

Patients (N = 6528) took colchicine 0.5 mg daily as part of a 1-month, open-label run-in phase; 1006 patients stopped taking colchicine during this time. Perceived adverse effects were observed in 611 of these patients, the most common being gastrointestinal (GI) upset (437 patients). After the run-in phase, the remaining 5522 patients were randomized to either the colchicine or placebo group. Both groups continued to receive GDMT for CVD, including antiplatelet therapy, anticoagulants, and hypertensive therapy as indicated. Lipid-lowering therapies were continued in 96.7% of the colchicine group and 96.6% of the placebo group. These patients were then followed for a minimum of 1 year (median duration, 28.6 months).

Continue to: The primary endpoint...

 

 

The primary endpoint occurred less frequently in the colchicine group than in the placebo group (6.8% vs 9.6%; P < .001; number needed to treat = 36). The incidence rates for 2 of the individual outcomes in the composite, MI (hazard ratio [HR] = 0.7; 95% CI, 0.53-0.93) and ischemia-driven coronary revascularization (HR = 0.75; 95% CI, 0.60-0.94), were significantly lower in the colchicine group. The other outcomes were no different from placebo.1

There was a similar incidence of serious adverse events, such as noncardiovascular death, cancer diagnosis, and hospitalization for infection, pneumonia, or GI issues. High-dose statins were used by 3413 patients (61.8%). Myalgia (data collected only from the Netherlands cohort) was reported more commonly in the colchicine group than the placebo group (21.2% vs 18.5%; cumulative incidence ratio = 1.15; 95% CI, 1.01-1.31). Myotoxic effects were rare in both groups.1

WHAT’S NEW

RCT supports potential for anti-inflammatory therapy in CAD

This large RCT demonstrated that the addition of daily colchicine reduces CVE risk in patients with known CAD while maintaining a good safety profile.1

CAVEATS

Watch for potential drug interactions in patients with renal dysfunction

Prescribers should be aware of potential drug interactions, especially in those with renal or hepatic dysfunction, when prescribing colchicine, as it is metabolized through cytochrome P450 3A4 (CYP3A4) and excreted via the P-glycoprotein transport system, by which many statins are also metabolized and act as a competitive substrate.7 In addition, simvastatin, and to a lesser degree atorvastatin, are CYP3A4 inhibitors.

Also of note, the 0.5-mg colchicine tablet is not available in some countries—including the United States, where only 0.6-mg tablets are available. The 0.6-mg dose would likely have the same benefit and similar adverse effect profile but was not included in the study.

CHALLENGES TO IMPLEMENTATION

GI tolerability may be an issue

Colchicine is widely available and relatively low in cost, at approximately $32 per month for the 0.6-mg daily tablets. A major limitation is lack of tolerability, as adverse effects such as nausea, vomiting, diarrhea, and abdominal pain are frequently reported.

Files
References

1. Nidorf SM, Fiolet ATL, Mosterd A, et al; LoDoCo2 Trial Investigators. Colchicine in patients with chronic coronary disease. N Engl J Med. 2020;383:1838-1847. doi: 10.1056/NEJMoa2021372

2. Laslett LJ, Alagona P Jr, Clark BA III, et al. The worldwide environment of cardiovascular disease: prevalence, diagnosis, therapy, and policy issues: a report from the American College of Cardiology. J Am Coll Cardiol. 2012;60(suppl):S1-S49. doi: 10.1016/j.jacc.2012.11.002

3. Bhatt DL, Eagle KA, Ohman EM, et al; REACH Registry Investigators. Comparative determinants of 4-year cardiovascular event rates in stable outpatients at risk of or with atherothrombosis. JAMA. 2010;304:1350-1357. doi: 10.1001/jama.2010.13224. Hansson GK. Inflammation, atherosclerosis, and coronary artery disease. N Engl J Med. 2005;352:1685-1695. doi: 10.1056/NEJMra043430

5. Ridker PM, Everett BM, Thuren T, et al; CANTOS Trial Group. Antiinflammatory therapy with canakinumab for atherosclerotic disease. N Engl J Med. 2017;377:1119-1131. doi: 10.1056/NEJMoa1707914

6. Imazio M, Bobbio M, Cecchi E, et al. Colchicine in addition to conventional therapy for acute pericarditis: results of the COlchicine for acute PEricarditis (COPE) trial. Circulation. 2005;112:2012-2016. doi: 10.1161/CIRCULATIONAHA.105.542738

7. Angelidis C, Kotsialou Z, Kossyvakis C, et al. Colchicine pharmacokinetics and mechanism of action. Curr Pharm Des. 2018;24:659-663. doi: 10.2174/1381612824666180123110042

8. Martínez GJ, Celermajer DS, Patel S. The NLRP3 inflammasome and the emerging role of colchicine to inhibit atherosclerosis-associated inflammation. Atherosclerosis. 2018;269:262-271. doi: 10.1016/j.atherosclerosis.2017.12.027

9. Nidorf SM, Eikelboom JW, Budgeon CA, et al. Low-dose colchicine for secondary prevention of cardiovascular disease. J Am Coll Cardiol. 2013;61:404-410. doi: 10.1016/j.jacc.2012.10.027

10. Tardif JC, Kouz S, Waters DD, et al. Efficacy and safety of low-dose colchicine after myocardial infarction. N Engl J Med. 2019;381:2497-2505. doi: 10.1056/NEJMoa1912388

Article PDF
Author and Disclosure Information

University of Tennessee College of Medicine, Chattanooga

DEPUTY EDITOR
Corey Lyon, DO

University of Colorado, Denver

Issue
The Journal of Family Practice - 71(6)
Publications
Topics
Page Number
E1-E3
Sections
Files
Files
Author and Disclosure Information

University of Tennessee College of Medicine, Chattanooga

DEPUTY EDITOR
Corey Lyon, DO

University of Colorado, Denver

Author and Disclosure Information

University of Tennessee College of Medicine, Chattanooga

DEPUTY EDITOR
Corey Lyon, DO

University of Colorado, Denver

Article PDF
Article PDF

ILLUSTRATIVE CASE

A 62-year-old woman with a past medical history of type 2 diabetes, hyperlipidemia, hypertension, and remote myocardial infarction (MI) presents to her primary care office for a preventive visit. She is a nonsmoker and has been taking her daily medications as prescribed, including an angiotensin-converting enzyme inhibitor, high-intensity statin, and aspirin. Her diabetes is well controlled. What else would you consider recommending to decrease this patient’s risk for future CVEs?

Cardiovascular disease (CVD) is a major contributor to morbidity and mortality, affecting more than 50% of patients older than 60.2 Despite control of risk factors with standard treatment modalities, patients with established CVD remain at high risk for future events, which makes elucidating and targeting other causative pathways essential.3

Inflammation has been identified as a key player in the development and progression of atherosclerosis and its downstream effects, with increased inflammatory markers correlating with increased risk for CVEs.4 Due to these findings, anti-inflammatory treatments have been under investigation as agents to further reduce risk for CVEs. In 1 such trial, the Canakinumab Antiinflammatory Thrombosis Outcome Study (CANTOS), patients with MI and elevated C-reactive protein levels treated with the interleukin-1 beta inhibitor canakinumab showed reduced risk for future CVEs compared to those receiving placebo.5 However, due to canakinumab’s high cost, inconvenient subcutaneous administration, and increased incidence of fatal infections, other agents are under investigation.

Colchicine is a potent anti-inflammatory agent, with approval in the United States for treatment of gout and familial Mediterranean fever. It works broadly to reduce inflammation by disrupting tubulin polymerization.6,7 Colchicine decreases interleukin-1 beta production through inactivation of the NLRP3 inflammasome pathway, which has been associated with the inflammatory component driving atherosclerotic plaque progression and instability.5,8 Colchicine’s oral administration, relative cost-effectiveness, and safety profile make it an attractive option for potential use in secondary prevention of CVEs.

The Low-Dose Colchicine (LoDoCo) trial, published in 2013, demonstrated a reduction in CVEs in those with CVD taking guideline-directed medical therapy (GDMT) plus colchicine 0.5 mg/d, compared with those taking GDMT alone.9 However, the LoDoCo study enrolled only 532 patients and was not placebo controlled. The Colchicine Cardiovascular Outcomes Trial (COLCOT), published in 2019, was a randomized, ­double-blind, placebo-controlled trial that aimed to further evaluate the effects of colchicine on CVEs on a larger scale and to assess its longer-term safety.10 In this study, the colchicine group had a significantly lower risk of CVEs vs placebo, with a comparable safety profile.10

STUDY SUMMARY

Fewer CVEs occurred when colchicine was added to the regimen

The randomized, multicenter, double-blind Low Dose Colchicine 2 (LoDoCo2) trial evaluated whether colchicine 0.5 mg daily reduces CV death, spontaneous (nonprocedural) MI, ischemic stroke, or ischemia-driven coronary revascularization in patients with chronic CAD (composite primary endpoint). This trial included 5522 patients, ages 35 to 82, in Australia and the Netherlands. Patients were eligible to participate if they had evidence of CAD by invasive coronary angiography, coronary calcium score, or computed tomography angiography, as well as evidence of clinical stability for 6 months. Exclusion criteria included moderate-to-severe renal impairment, severe heart failure, severe valvular disease, or intolerance to colchicine.

Patients (N = 6528) took colchicine 0.5 mg daily as part of a 1-month, open-label run-in phase; 1006 patients stopped taking colchicine during this time. Perceived adverse effects were observed in 611 of these patients, the most common being gastrointestinal (GI) upset (437 patients). After the run-in phase, the remaining 5522 patients were randomized to either the colchicine or placebo group. Both groups continued to receive GDMT for CVD, including antiplatelet therapy, anticoagulants, and hypertensive therapy as indicated. Lipid-lowering therapies were continued in 96.7% of the colchicine group and 96.6% of the placebo group. These patients were then followed for a minimum of 1 year (median duration, 28.6 months).

Continue to: The primary endpoint...

 

 

The primary endpoint occurred less frequently in the colchicine group than in the placebo group (6.8% vs 9.6%; P < .001; number needed to treat = 36). The incidence rates for 2 of the individual outcomes in the composite, MI (hazard ratio [HR] = 0.7; 95% CI, 0.53-0.93) and ischemia-driven coronary revascularization (HR = 0.75; 95% CI, 0.60-0.94), were significantly lower in the colchicine group. The other outcomes were no different from placebo.1

There was a similar incidence of serious adverse events, such as noncardiovascular death, cancer diagnosis, and hospitalization for infection, pneumonia, or GI issues. High-dose statins were used by 3413 patients (61.8%). Myalgia (data collected only from the Netherlands cohort) was reported more commonly in the colchicine group than the placebo group (21.2% vs 18.5%; cumulative incidence ratio = 1.15; 95% CI, 1.01-1.31). Myotoxic effects were rare in both groups.1

WHAT’S NEW

RCT supports potential for anti-inflammatory therapy in CAD

This large RCT demonstrated that the addition of daily colchicine reduces CVE risk in patients with known CAD while maintaining a good safety profile.1

CAVEATS

Watch for potential drug interactions in patients with renal dysfunction

Prescribers should be aware of potential drug interactions, especially in those with renal or hepatic dysfunction, when prescribing colchicine, as it is metabolized through cytochrome P450 3A4 (CYP3A4) and excreted via the P-glycoprotein transport system, by which many statins are also metabolized and act as a competitive substrate.7 In addition, simvastatin, and to a lesser degree atorvastatin, are CYP3A4 inhibitors.

Also of note, the 0.5-mg colchicine tablet is not available in some countries—including the United States, where only 0.6-mg tablets are available. The 0.6-mg dose would likely have the same benefit and similar adverse effect profile but was not included in the study.

CHALLENGES TO IMPLEMENTATION

GI tolerability may be an issue

Colchicine is widely available and relatively low in cost, at approximately $32 per month for the 0.6-mg daily tablets. A major limitation is lack of tolerability, as adverse effects such as nausea, vomiting, diarrhea, and abdominal pain are frequently reported.

ILLUSTRATIVE CASE

A 62-year-old woman with a past medical history of type 2 diabetes, hyperlipidemia, hypertension, and remote myocardial infarction (MI) presents to her primary care office for a preventive visit. She is a nonsmoker and has been taking her daily medications as prescribed, including an angiotensin-converting enzyme inhibitor, high-intensity statin, and aspirin. Her diabetes is well controlled. What else would you consider recommending to decrease this patient’s risk for future CVEs?

Cardiovascular disease (CVD) is a major contributor to morbidity and mortality, affecting more than 50% of patients older than 60.2 Despite control of risk factors with standard treatment modalities, patients with established CVD remain at high risk for future events, which makes elucidating and targeting other causative pathways essential.3

Inflammation has been identified as a key player in the development and progression of atherosclerosis and its downstream effects, with increased inflammatory markers correlating with increased risk for CVEs.4 Due to these findings, anti-inflammatory treatments have been under investigation as agents to further reduce risk for CVEs. In 1 such trial, the Canakinumab Antiinflammatory Thrombosis Outcome Study (CANTOS), patients with MI and elevated C-reactive protein levels treated with the interleukin-1 beta inhibitor canakinumab showed reduced risk for future CVEs compared to those receiving placebo.5 However, due to canakinumab’s high cost, inconvenient subcutaneous administration, and increased incidence of fatal infections, other agents are under investigation.

Colchicine is a potent anti-inflammatory agent, with approval in the United States for treatment of gout and familial Mediterranean fever. It works broadly to reduce inflammation by disrupting tubulin polymerization.6,7 Colchicine decreases interleukin-1 beta production through inactivation of the NLRP3 inflammasome pathway, which has been associated with the inflammatory component driving atherosclerotic plaque progression and instability.5,8 Colchicine’s oral administration, relative cost-effectiveness, and safety profile make it an attractive option for potential use in secondary prevention of CVEs.

The Low-Dose Colchicine (LoDoCo) trial, published in 2013, demonstrated a reduction in CVEs in those with CVD taking guideline-directed medical therapy (GDMT) plus colchicine 0.5 mg/d, compared with those taking GDMT alone.9 However, the LoDoCo study enrolled only 532 patients and was not placebo controlled. The Colchicine Cardiovascular Outcomes Trial (COLCOT), published in 2019, was a randomized, ­double-blind, placebo-controlled trial that aimed to further evaluate the effects of colchicine on CVEs on a larger scale and to assess its longer-term safety.10 In this study, the colchicine group had a significantly lower risk of CVEs vs placebo, with a comparable safety profile.10

STUDY SUMMARY

Fewer CVEs occurred when colchicine was added to the regimen

The randomized, multicenter, double-blind Low Dose Colchicine 2 (LoDoCo2) trial evaluated whether colchicine 0.5 mg daily reduces CV death, spontaneous (nonprocedural) MI, ischemic stroke, or ischemia-driven coronary revascularization in patients with chronic CAD (composite primary endpoint). This trial included 5522 patients, ages 35 to 82, in Australia and the Netherlands. Patients were eligible to participate if they had evidence of CAD by invasive coronary angiography, coronary calcium score, or computed tomography angiography, as well as evidence of clinical stability for 6 months. Exclusion criteria included moderate-to-severe renal impairment, severe heart failure, severe valvular disease, or intolerance to colchicine.

Patients (N = 6528) took colchicine 0.5 mg daily as part of a 1-month, open-label run-in phase; 1006 patients stopped taking colchicine during this time. Perceived adverse effects were observed in 611 of these patients, the most common being gastrointestinal (GI) upset (437 patients). After the run-in phase, the remaining 5522 patients were randomized to either the colchicine or placebo group. Both groups continued to receive GDMT for CVD, including antiplatelet therapy, anticoagulants, and hypertensive therapy as indicated. Lipid-lowering therapies were continued in 96.7% of the colchicine group and 96.6% of the placebo group. These patients were then followed for a minimum of 1 year (median duration, 28.6 months).

Continue to: The primary endpoint...

 

 

The primary endpoint occurred less frequently in the colchicine group than in the placebo group (6.8% vs 9.6%; P < .001; number needed to treat = 36). The incidence rates for 2 of the individual outcomes in the composite, MI (hazard ratio [HR] = 0.7; 95% CI, 0.53-0.93) and ischemia-driven coronary revascularization (HR = 0.75; 95% CI, 0.60-0.94), were significantly lower in the colchicine group. The other outcomes were no different from placebo.1

There was a similar incidence of serious adverse events, such as noncardiovascular death, cancer diagnosis, and hospitalization for infection, pneumonia, or GI issues. High-dose statins were used by 3413 patients (61.8%). Myalgia (data collected only from the Netherlands cohort) was reported more commonly in the colchicine group than the placebo group (21.2% vs 18.5%; cumulative incidence ratio = 1.15; 95% CI, 1.01-1.31). Myotoxic effects were rare in both groups.1

WHAT’S NEW

RCT supports potential for anti-inflammatory therapy in CAD

This large RCT demonstrated that the addition of daily colchicine reduces CVE risk in patients with known CAD while maintaining a good safety profile.1

CAVEATS

Watch for potential drug interactions in patients with renal dysfunction

Prescribers should be aware of potential drug interactions, especially in those with renal or hepatic dysfunction, when prescribing colchicine, as it is metabolized through cytochrome P450 3A4 (CYP3A4) and excreted via the P-glycoprotein transport system, by which many statins are also metabolized and act as a competitive substrate.7 In addition, simvastatin, and to a lesser degree atorvastatin, are CYP3A4 inhibitors.

Also of note, the 0.5-mg colchicine tablet is not available in some countries—including the United States, where only 0.6-mg tablets are available. The 0.6-mg dose would likely have the same benefit and similar adverse effect profile but was not included in the study.

CHALLENGES TO IMPLEMENTATION

GI tolerability may be an issue

Colchicine is widely available and relatively low in cost, at approximately $32 per month for the 0.6-mg daily tablets. A major limitation is lack of tolerability, as adverse effects such as nausea, vomiting, diarrhea, and abdominal pain are frequently reported.

References

1. Nidorf SM, Fiolet ATL, Mosterd A, et al; LoDoCo2 Trial Investigators. Colchicine in patients with chronic coronary disease. N Engl J Med. 2020;383:1838-1847. doi: 10.1056/NEJMoa2021372

2. Laslett LJ, Alagona P Jr, Clark BA III, et al. The worldwide environment of cardiovascular disease: prevalence, diagnosis, therapy, and policy issues: a report from the American College of Cardiology. J Am Coll Cardiol. 2012;60(suppl):S1-S49. doi: 10.1016/j.jacc.2012.11.002

3. Bhatt DL, Eagle KA, Ohman EM, et al; REACH Registry Investigators. Comparative determinants of 4-year cardiovascular event rates in stable outpatients at risk of or with atherothrombosis. JAMA. 2010;304:1350-1357. doi: 10.1001/jama.2010.13224. Hansson GK. Inflammation, atherosclerosis, and coronary artery disease. N Engl J Med. 2005;352:1685-1695. doi: 10.1056/NEJMra043430

5. Ridker PM, Everett BM, Thuren T, et al; CANTOS Trial Group. Antiinflammatory therapy with canakinumab for atherosclerotic disease. N Engl J Med. 2017;377:1119-1131. doi: 10.1056/NEJMoa1707914

6. Imazio M, Bobbio M, Cecchi E, et al. Colchicine in addition to conventional therapy for acute pericarditis: results of the COlchicine for acute PEricarditis (COPE) trial. Circulation. 2005;112:2012-2016. doi: 10.1161/CIRCULATIONAHA.105.542738

7. Angelidis C, Kotsialou Z, Kossyvakis C, et al. Colchicine pharmacokinetics and mechanism of action. Curr Pharm Des. 2018;24:659-663. doi: 10.2174/1381612824666180123110042

8. Martínez GJ, Celermajer DS, Patel S. The NLRP3 inflammasome and the emerging role of colchicine to inhibit atherosclerosis-associated inflammation. Atherosclerosis. 2018;269:262-271. doi: 10.1016/j.atherosclerosis.2017.12.027

9. Nidorf SM, Eikelboom JW, Budgeon CA, et al. Low-dose colchicine for secondary prevention of cardiovascular disease. J Am Coll Cardiol. 2013;61:404-410. doi: 10.1016/j.jacc.2012.10.027

10. Tardif JC, Kouz S, Waters DD, et al. Efficacy and safety of low-dose colchicine after myocardial infarction. N Engl J Med. 2019;381:2497-2505. doi: 10.1056/NEJMoa1912388

References

1. Nidorf SM, Fiolet ATL, Mosterd A, et al; LoDoCo2 Trial Investigators. Colchicine in patients with chronic coronary disease. N Engl J Med. 2020;383:1838-1847. doi: 10.1056/NEJMoa2021372

2. Laslett LJ, Alagona P Jr, Clark BA III, et al. The worldwide environment of cardiovascular disease: prevalence, diagnosis, therapy, and policy issues: a report from the American College of Cardiology. J Am Coll Cardiol. 2012;60(suppl):S1-S49. doi: 10.1016/j.jacc.2012.11.002

3. Bhatt DL, Eagle KA, Ohman EM, et al; REACH Registry Investigators. Comparative determinants of 4-year cardiovascular event rates in stable outpatients at risk of or with atherothrombosis. JAMA. 2010;304:1350-1357. doi: 10.1001/jama.2010.13224. Hansson GK. Inflammation, atherosclerosis, and coronary artery disease. N Engl J Med. 2005;352:1685-1695. doi: 10.1056/NEJMra043430

5. Ridker PM, Everett BM, Thuren T, et al; CANTOS Trial Group. Antiinflammatory therapy with canakinumab for atherosclerotic disease. N Engl J Med. 2017;377:1119-1131. doi: 10.1056/NEJMoa1707914

6. Imazio M, Bobbio M, Cecchi E, et al. Colchicine in addition to conventional therapy for acute pericarditis: results of the COlchicine for acute PEricarditis (COPE) trial. Circulation. 2005;112:2012-2016. doi: 10.1161/CIRCULATIONAHA.105.542738

7. Angelidis C, Kotsialou Z, Kossyvakis C, et al. Colchicine pharmacokinetics and mechanism of action. Curr Pharm Des. 2018;24:659-663. doi: 10.2174/1381612824666180123110042

8. Martínez GJ, Celermajer DS, Patel S. The NLRP3 inflammasome and the emerging role of colchicine to inhibit atherosclerosis-associated inflammation. Atherosclerosis. 2018;269:262-271. doi: 10.1016/j.atherosclerosis.2017.12.027

9. Nidorf SM, Eikelboom JW, Budgeon CA, et al. Low-dose colchicine for secondary prevention of cardiovascular disease. J Am Coll Cardiol. 2013;61:404-410. doi: 10.1016/j.jacc.2012.10.027

10. Tardif JC, Kouz S, Waters DD, et al. Efficacy and safety of low-dose colchicine after myocardial infarction. N Engl J Med. 2019;381:2497-2505. doi: 10.1056/NEJMoa1912388

Issue
The Journal of Family Practice - 71(6)
Issue
The Journal of Family Practice - 71(6)
Page Number
E1-E3
Page Number
E1-E3
Publications
Publications
Topics
Article Type
Display Headline
Colchicine may decrease cardiovascular events in patients with coronary artery disease
Display Headline
Colchicine may decrease cardiovascular events in patients with coronary artery disease
Sections
PURLs Copyright
Copyright © 2022. The Family Physicians Inquiries Network. All rights reserved.
Inside the Article

PRACTICE CHANGER

Consider prescribing colchicine 0.5 mg daily as an addition to current standard-of-care therapies for patients with coronary artery disease (CAD) to prevent further cardiovascular events (CVEs).

STRENGTH OF RECOMMENDATION

B: Based on a single randomized controlled trial (RCT).1

Nidorf SM, Fiolet ATL, Mosterd A, et al; LoDoCo2 Trial Investigators. Colchicine in patients with chronic coronary disease. N Engl J Med. 2020;383:1838-1847.

Disallow All Ads
Content Gating
No Gating (article Unlocked/Free)
Alternative CME
Disqus Comments
Default
Use ProPublica
Hide sidebar & use full width
render the right sidebar.
Conference Recap Checkbox
Not Conference Recap
Clinical Edge
Display the Slideshow in this Article
Medscape Article
Display survey writer
Reuters content
Disable Inline Native ads
WebMD Article
Article PDF Media
Media Files